SlideShare a Scribd company logo
1 of 57
Bile
Lyndon Woytuck, BSc, MBBS student
What is bile?
 Bile is a fluid made by liver cells (hepatocytes),
secreted into the biliary tract and stored in the
gallbladder.
 Bile aids in digestion by breaking down fats into fatty
acids, which can be absorbed by the digestive tract for
further use by the body.
 Bile is mainly composed of cholesterol, bile acids (or
bile salts), and bilirubin (a breakdown product of red
blood cells). Bile also contains water, potassium and
sodium salts, copper and trace metals
A note on the Anatomy
 The common hepatic duct then merges with the cystic duct from the
gallbladder to form the common bile duct.
 The common bile duct joins with the duodenum at the Ampulla of
Vater.
 About half of the bile produced by the liver is first stored in the
gallbladder before emptying into the duodenum through the
sphincter of Oddi.
 The gallbladder is a pear-shaped organ attached beneath the liver.
 When food enters the duodenum, the gallbladder contracts and
releases stored bile into the intestinal lumen to aid digestion.
 Bile secreted from hepatocytes is collected by a
system of ducts flowing within the liver and
eventually draining into the left and right
hepatic ducts.
 The hepatic ducts drain into the common hepatic
duct.
Naturally, the physiology
 The biliary system drains waste products from the liver into
the duodenum,
 And aids in digestion via CCK controlled release of bile
 Bile is a greenish-yellow fluid and serves two primary
functions:
 Excretion of waste
 Fat digestion
 Bile acids emulsify and absorb fats with vitamins A, D, E, K
 Stercobilin is excreted from the body in faeces, and is what
gives faeces its dark brown color.
 Urobilin is carried through the blood plasma and excreted
in urine, giving urine a yellow (sometimes brown) colour
For the image shown, answer the following questions:
1. Identify the labeled structures (A-G).
2. What is the name of the sphincter that controls the
structure labeled (L)?
3. What life-threatening complication can develop as a
result of a gallstone becoming lodged within the
structure labeled (L)?
4. BONUS: Which hormone stimulates the release of bile
from the gallbladder?
Answer:
1. (A) Right hepatic duct, (B) left hepatic duct, (C) cystic duct, (D)
common hepatic duct, (E) common bile duct, (F) accessory pancreatic
duct, and (G) pancreatic duct.
2. The sphincter responsible for controlling the major duodenal papilla
(ampulla of Vater) (L) is the sphincter of Oddi.
3. Gallstone-induced pancreatitis can result from a gallstone that becomes
lodged within the major duodenal papilla (ampulla of Vater) (L).
4. Cholecystokinin (CCK), which is produced by I cells of the duodenum and
jejunum, stimulates the gallbladder to contract and release bile.
Explain how disruption of
bilirubin metabolism and
excretion can cause jaundice
 Jaundice is caused by an accumulation of bile pigments (biliverdin,
bilirubin) in the body. This is usually first visible in mucus
membranes (sclerae), then followed by the skin.
 Haemolytic
 increased haemoglobin breakdown produces more bilirubin, which
overloads the conjugation. Increase in unconjugated form. Other
LFTs usually normal, with reduced haptoglobin, reticulocytosis and
abnormal red cells on film. Anaemia may occur.
 Hepatocellular
 failure of conjugation resulting in increased bilirubin. Increased ALT
(marker of hepatocellular damage). Damage to hepatocytes may
cause leakage of bilirubin into plasma and conjugated bilirubin may
be found in the urine.
 Obstructive
 biliary obstruction results in conjugated bilirubin to pass from the
liver to plasma. Very high levels may cause the patient to turn
green and turn the urine dark. Less stercobilin and faeces turn
pale. Elevated ALP due to cholestasis.
Carotenaemia or quinacrine
ingestion can result in yellow or
green skin colour precipitated by
eating large amounts of green
and yellow vegetables, corn or
tomatoes. In absence of yellow
mucous membranes and sclerae,
with normal urine color, and
accentuated yellow-brown
pigment on palms, soles, and
nasolabial folds. Quinacrine used
for treatment of giardiasis may
colour skin yellow, but with
normal urine. Serum bilirubin
levels are normal.
Describe the pathogenesis of conjugated
and unconjugated hyperbilirubinaemia and
list conditions associated with each
 Conjugated:
 Obstructive – cholestasis, cholangitis
 Hepatocellular – infection, hepatocellular carcinoma, cirrhosis
 Physiologic jaundice of the newborn
 Breast milk jaundice
 Dubin-Johnson's / Rotor's: impairment of hepatocyte bilirubin secretion
 Unconjugated:
 Haemolytic – haemolytic disease of the newborn (Rh), Wilson’s,
autoimmune hepatitis, alcoholic, drug induced
 Ineffective erythropoiesis
 Gilbert's: hepatocyte bilirubin uptake alteration
 Crigler-Najjar's: glucuronyl transferase deficiency
 Lab report of liver function tests of a patient with
jaundice showed predominantly conjugated
hyperbilirubinaemia. Which of the following conditions
is most likely to be responsible for conjugated
hyperbilirubinaemia?
A. Acute haemolytic crisis in sickle cell disease
B. Gilbert syndrome
C. Haemolysis due to rhesus incompatibility
D. Obstructive jaundice due to carcinoma of common bile
duct
E. Physiological jaundice of the newborn
 Lab report of liver function tests of a patient with
jaundice showed predominantly conjugated
hyperbilirubinaemia. Which of the following conditions
is most likely to be responsible for conjugated
hyperbilirubinaemia?
A. Acute haemolytic crisis in sickle cell disease
B. Gilbert syndrome
C. Haemolysis due to rhesus incompatibility
D. Obstructive jaundice due to carcinoma of common bile
duct
E. Physiological jaundice of the newborn
Describe the symptoms and signs in
a patient presenting with jaundice
 Yellow colouring of the skin and eyes (usually beginning
on the face and moving down the body)
 Direct and indirect bilirubin levels. These reflect
whether the bilirubin is bound with other substances by
the liver so that it can be excreted (direct), or is
circulating in the blood circulation (indirect). Either
measurement may be high.
 Red blood cell counts. Haemolytic jaundice may
present with anaemia or reticulocytosis.
Explain how features of the history and examination
can be used to distinguish between haemolytic,
hepatocellular or cholestatic jaundice
 Onset
 Few days, one week – hepatitis: viral, bacterial, drug or toxin
 Weeks – subacute hepatitis, extrahepatic obstruction
 Fluctuating intensity – cholelithiasis, ampullary cancer, drug hepatitis
 Past history – chronic hepatitis, cirrhosis, benign recurrent intrahepatic
cholestasis, genetic non-haemolytic hyperbilirubinaemia
 Age
 >40 pancreatic carcinoma, cholelithiasis
 <25 Hepatitis A
 Constitutional symptoms – anorexia, nausea, emesis, weight loss
 within 2 weeks prior: hepatitis or cholelithiasis
 Continuously >2 weeks: malignant biliary obstruction, chronic hepatitis,
toxin (alcohol)
 Recurrent brief episodes over months/years and RUQ pain:
cholelithiasis
Features of History and Exam:
Symptoms
 Abdominal pain
 Dull ache RUQ: acute hepatitis
 Acute abdomen (fever, jaundice and leucocytosis): alcoholic
hepatitis
 RUQ episodically with radiation to right scapula or girdle
distribution: cholelithiasis
 Epigastric or RUQ with radiation to back: pancreatic head
carcinoma
 Fever
 Acute hepatitis or with chills, biliary obstruction (stones or
stricture >> malignancy)
 Pruritis
 Biliary tract obstruction, occasionally viral hepatitis
 Recent onset: large ducts – neoplasm, or canaliculi – intrahepatic
(drug)
 Long standing, middle-aged woman: primary biliary cirrhosis
Features of History and Examination:
Portals of entry
 Drug and toxin exposures
 Pain relievers, tranquilizers, oestrogens, chemicals, alcohol
 Hepatic infection (viral, etc.)
 Patients, transfusions, needles, narcotics, raw shellfish, travel
(entamoeba), sexual, animals or stagnant water
(leptospirosis), immunocompromise, infectious mono, sepsis
 Surgery (<3 weeks)
 Increased bilirubin load: transfusion haemolysis, haematoma
resorption and G6PDd, drug reactions, malarial transfusion
 Impaired hepatic function: halogenated anaesthesia, drugs,
sepsis, hepatic ischaemia
 Obstruction: surgical injury, biliary calculus, cholecystitis
 Previous biliary surgery (<2 years): biliary stricture
Features of History and Examination:
Historical insights
 Systemic conditions:
 Inflammatory bowel disease
 primary sclerosing cholangitis, cholangiocarcinoma, chronic hepatitis, cirrhosis,
hepatic amyloid. Resection – cholelithiasis
 Cirrhosis
 Cystic fibrosis, haemochromatosis, Wilson’s, alpha-1 antitrypsin def.,
galactosaemia, hereditary fructose intolerance, tyrosinaemia
 Systemic lupus erythematosus
 Hypoxic injury – hepatitis
 Family history of jaundice or hepatic disease
 Genetically transmitted non-haemolytic hyperbilirubinaemias: Crigler-
Najjar, Gilbert’s, Dubin-Johnson, Rotor’s
 Benign recurrent intrahepatic cholestasis, Wilson’s disease,
haemochromatosis, alpha-1 antitrypsin def, hereditary spherocytosis
Features of History and Examination:
Physical Inspection
 Hepatocellular diseased patient appears more acutely ill
than obstructive disease
 Greenish jaundice – prolonged obstruction
 Orange-yellow jaundice – hepatocellular
 Mental derangements – hepatocellular >> obstruction
 Spider telangiectasias: chronic hepatocellular
 Scratch marks: pruritis
 Decreased axillary/pubic hair, gynaecoid changes: cirrhosis
 Dupuytren’s contracture: chronic liver disease
 Xanthelasma, tuberous xanthomas: long standing biliary
obstruction with hyperlipidaemia, primary biliary cirrhosis
Features of History and Examination:
Physical Auscultation, Palpation
 Hepatic bruits
 malignancy, hepatitis, haemangioma
 Hepatic friction rub
 malignancy, inflammatory (Glisson’s capsule)
 Standing up may aid here
 Very large liver
 congested or fatty cirrhosis, neoplasm or amyloid
 Rapidly shrinking liver
 acute liver failure – viral or toxin
 Hard or nodular
 fibrotic or malignant infiltration
Features of History and Examination:
Palpation, Percussion
 Unusual tenderness
 acute hepatitis, abscess or rapid enlargement (vascular
congestion or fatty changes)
 Splenomegaly
 w/out hepatomegaly: primary haemolytic or portal vein
occlusion
 Portal hypertension, viral hepatitis
 w/ hepatomegaly: malignancy (haematologic) or storage
disease
 Ascites
 cirrhosis with portal hypertension or malignancy >> massive or
subacute hepatic necrosis or hepatic vein obstruction
 A 52 year old man, who was previously well, presented with a 3
week history of increasing jaundice associated with pale stools,
dark urine and itching. There was no associated pain or fever. He
was not on any medication. Blood tests confirmed a cholestatic
pattern of hyperbilirubinaemia with markedly raised alkaline
phosphatase, moderate elevation of transaminases and normal
serum albumin. Regarding the planning of his further investigation,
which is true
A. Absence of pain excludes gallstone obstruction of the common bile
duct (CBD)
B. Absence of bile duct dilatation on ultrasound scan makes it safe to
immediately proceed to liver biopsy
C. Magnetic resonance cholangio-pancreatography (MRCP) is inferior
to endoscopic retrograde cholangio-pancreatography (ERCP) in the
investigation of extrahepatic cholestasis where ultrasonography
fails to show the cause
D. The role of ERCP is primarily to undertake therapeutic measures,
which might avoid surgical intervention
E. A prolonged prothrombin time due to extra-hepatic cholestasis
requires a 5 day course of intravenous vitamin K (5mg) to correct
 A 52 year old man, who was previously well, presented with a 3
week history of increasing jaundice associated with pale stools,
dark urine and itching. There was no associated pain or fever. He
was not on any medication. Blood tests confirmed a cholestatic
pattern of hyperbilirubinaemia with markedly raised alkaline
phosphatase, moderate elevation of transaminases and normal
serum albumin. Regarding the planning of his further investigation,
which is true
A. Absence of pain excludes gallstone obstruction of the common bile
duct (CBD)
B. Absence of bile duct dilatation on ultrasound scan makes it safe to
immediately proceed to liver biopsy
C. Magnetic resonance cholangio-pancreatography (MRCP) is inferior
to endoscopic retrograde cholangio-pancreatography (ERCP) in the
investigation of extrahepatic cholestasis where ultrasonography
fails to show the cause
D. The role of ERCP is primarily to undertake therapeutic
measures, which might avoid surgical intervention
E. A prolonged prothrombin time due to extra-hepatic cholestasis
requires a 5 day course of intravenous vitamin K (5mg) to correct
 Correct answer – D (www.medicinecpd.co.uk)
Explanation: Absence of pain does not exclude the
passage of gallstones obstructing the CBD; even the
appearance of bile duct dilatation may not be obvious
on USG if the obstruction is intermittent. In this
situation MRCP should be undertaken and the more
invasive ERCP reserved for the therapeutic procedures
such as stenting, sphincterotomy or stone extraction.
With a 3 week history of cholestatic jaundice
malabsorption of vitamin K is likely to have led to
prothrombin time (INR) prolongation and it would be
highly dangerous to undertake liver biopsy without first
correcting it. The INR should return to normal within 6
hours of a single intravenous dose of vitamin K (5mg).
Give a differential diagnosis for a
patient presenting with jaundice
 Benign recurrent intrahepatic
cholestasis
 Cholangitis – primary sclerosing
 Cholangiocarcinoma
 Cholecystitis
 Cholelithiasis
 Chronic pancreatitis
 Cirrhosis
 Common bile duct stricture
 Extrahepatic malignancy
 Genetic non-haemolytic
hyperbilirubinaemia (Gilbert’s
or Dubin-Johnson syndrome)
 Haemolysis
 HELLP syndrome (hemolysis,
elevated liver tests, and
thrombocytopenia)
 Hepatitis: viral, bacterial, drug
or toxin
 Hepatocellular carcinoma
 Hyperbilirubinemia of the
newborn
 Neonate – breast milk jaundice,
physiologic jaundice,
haemolytic disease
 Primary biliary cirrhosis
Take a structured history from a patient
with jaundice to determine aetiology
AmyJoanDice
•Presents to
ED
•Recent onset
upper
abdominal
discomfort
•Chills
•Yellow
sclerae
Whatshouldyouask? •Focused
HoPC
•Medications –
liver damage
•Alcohol or
IVDU
•Surgery –
biliary,
cancer
•Transfusions
•Pregnancy
•Occupation
Symptoms?
•Character
•Sequence
•Relievers or
conceivers
•Associated
signs/Sx
•Urine/stool
•PMHx, FHx
Explain the relevance of changes in colour and
bilirubin and urobilinogen content of stools and
urine in the assessment of jaundice
 Dark urine, with green foam upon shaking is caused by
bile pigment
 excludes hemolysis or a hepatic uptake or conjugating
defect of bilirubin metabolism acting alone
 Brown stool: haemolysis, mild to moderate
hepatocellular
 Clay coloured stool: moderate to severe hepatocellular,
obstructive
 Blood in clay-coloured stool: carcinoma of the pancreas
or ampulla of Vater.
 A 40 year old woman, who is jaundiced, presents to you
with reports of laboratory tests that reveal conjugated
hyperbilirubinaemia. Urine bilirubin levels are
significantly above normal while urine urobilinogen
levels are significantly below normal. Which of the
following is most likely cause of her jaundice?
A. Blockage of the common bile duct
B. Deficiency of glucuronyltransferase
C. Gilbert syndrome
D. Haemolytic anaemia
E. Primary shunt hyperbilirubinaemia
 A 40 year old woman, who is jaundiced, presents to you
with reports of laboratory tests that reveal conjugated
hyperbilirubinaemia. Urine bilirubin levels are
significantly above normal while urine urobilinogen
levels are significantly below normal. Which of the
following is most likely cause of her jaundice?
A. Blockage of the common bile duct
B. Deficiency of glucuronyltransferase
C. Gilbert syndrome
D. Haemolytic anaemia
E. Primary shunt hyperbilirubinaemia
Outline the mechanisms whereby drugs may
cause jaundice and give examples of drugs which
have each effect
 Hepatitis/Hepatotoxicity
 Acetaminophen, NSAIDs, Amiodarone, Anabolic steroids, Birth control pills, Chlorpromazine,
Erythromycin, Halothan, Methyldopa, Isoniazid, Methotrexate, Statins, Sulfa drugs,
Amoxicillin-clavulanate, Anti-epileptics
 Cholestasis
 Cyclosporine, bosentan, glibenclamide, troglitrazone, rifampicin
 Normally, the liver metabolises certain pharma
 Susceptible individuals may be poor metabolisers
 Hepatocellular uptake and biotransformation results in more water soluble metabolites from
lipid soluble drugs
 Phase I reactions involve the oxidation, hydroxylation and other reactions mediated by the
cytochrome P-450 (CYP) system, particularly CYP3A4. Activity of the cytochrome P-450
system varies greatly among individuals.
 Phase II reactions involve esterification reactions that form conjugates with sulfate,
glucuronic acid, amino acids or glutathione molecules - enhances detoxification of the
compounds.
 Can also lead to the production of toxic intermediates
 Drug induced cholestasis may happen with high drug concentrations, genetic alterations of
enzyme or transporter expression, and/or lower concentrations of anti-oxidants, such as
glutathione. Can be caused by direct toxic effects of drugs or their metabolites on liver cells
or through by immune mediation
Describe how you would investigate
a patient with jaundice
 Phlebotomy
1) Complete blood cell count and blood smear
1) Haemolysis, reticulocytosis, leukocytosis and neutrophilia, eosinophilia
2) In newborns, blood type and testing for Rh incompatibility
(Coomb's)
3) Urinalysis: bilirubin, protein
4) Conjugated and unconjugated (direct and indirect) bilirubin
5) Associated liver function tests (ALP, SGPT and SGOT)
6) Prothrombin and prothrombin precursor, prothrombin time –
prognostic
7) Albumin – chronicity
8) Hepatitis A, B, C serology
9) Antibody titres – ANA, delta agent, Epstein-Barr, herpes,
cytomegalovirus (leptospirosis, syphilis, entamoeba)
10) Specific markers: ceruloplasmin, transferrin sat, HFE, alpha
fetoprotein, protease inhibitors
Describe how you would investigate
a patient with jaundice
 Imaging
 Chest X-Ray, Abdomen
 Abdominal sonography
 CT
 Percutaneous transhepatic cholangiography (PTC)
 Endoscopic retrograde cholangiopancreatography (ERCP)
 Hepatobiliary scintigraphy (HBS)
 MRI: metastasis
 Biopsy
Describe the hepatic origins of ALP, AST and GGT
and explain how changes in these may indicate the
origin of the jaundice
 Alkaline phosphatase
 ALP = removes phophates – dephosphorylation
 cholestasis, hepatocellular enzyme induction, canalicular injury, bone growth or
disease, placenta
 Acute lithic biliary obstruction may have aminotransferase levels >500 U/L and
normal or mildly elevated ALP
 Aminotransferases
 AST = amino group catalysis – amino acid metabolism
 hepatocellular injury, acute biliary obstruction, coeliac disease, skeletal
muscle (AST)
 Progression and resolution of hepatocellular injury
 AST:ALT >2:1 may indicate alcoholic liver disease
 Gamma-glutamyl transpeptidase
 GGT catalyses glutathione to an acceptor – detox and glutamate cycle
 cholestasis, medications, ethanol, hyperthyroidism, myotonic dystrophy
 Jaundice is yellowing of the skin, sclerae and other
tissues caused by excess circulating bilirubin. Jaundice
is likely to be due to:
A. Common bile duct obstruction if the serum
aminotransferases are elevated and alkaline phosphatase
is low
B. Haemolytic disease if plasma albumin is low and globulin
high
C. Haemolytic disease if prothrombin time is prolonged
D. Hepatic disease if plasma albumin is low and serum
aminotransferase elevations >500 units
E. Hepatic disease if plasma acid phosphatase level is raised
 Jaundice is yellowing of the skin, sclerae and other
tissues caused by excess circulating bilirubin. Jaundice
is likely to be due to:
A. Common bile duct obstruction if the serum
aminotransferases are elevated and alkaline phosphatase
is low
B. Haemolytic disease if plasma albumin is low and globulin
high
C. Haemolytic disease if prothrombin time is prolonged
D. Hepatic disease if plasma albumin is low and serum
aminotransferase elevations >500 units
E. Hepatic disease if plasma acid phosphatase level is raised
Give a differential diagnosis for
extrahepatic biliary obstruction
 Cancer
 Cholangiocarcinoma
 pancreatic carcinoma
 Iatrogenic
 surgical injury, stricture, biliary leak
 Cholelithiasis
 Cholecystitis
 Cholangitis
Describe the investigation of a patient with extrahepatic
biliary obstruction, including endoscopic, radiological and
surgical techniques available for treatment/palliation of
the underlying condition
 Cholelithiasis
 Extract lodged stones from the common bile duct tree by performing a
procedure called ERCP (endoscopic retrograde cholangiopancreatography).
 Strictures
 surgical or by interventional endoscopy or radiology with possible further
operations to remove stone source
 Bile duct cancer
 Diagnosed by radiology, furthered by ERCP or PTC and EUS
 Resection of the cancer and pathologic exam
 Pancreatic cancer
 Diagnosed by radiology, multidisciplinary treatment
 Resection and biliary tree reconstruction
 Cholecystitis; Cholangitis
 Cure infection, then cure the underlying cause
 Among the following, which is the investigation of
choice for evaluation of ONLY the common bile duct?
A. CECT abdomen (computed tomography)
B. MRCP (magnetic resonance cholangio-pancreatography)
C. HIDA (hepatobiliary) scan
D. Ultrasonography
 Among the following, which is the investigation of
choice for evaluation of common bile duct?
A. CECT abdomen
B. MRCP (magnetic resonance cholangio-pancreatography)
C. HIDA (hepatobiliary) scan
D. Ultrasonography
Describe the symptoms of
biliary colic
 Pain
 1-5 hours of constant severe or dull pain in the epigastrium or RUQ
 Peritoneal irritation localises to RUQ
 May radiate to scapula or back
 Patients move around to seek pain relief
 Pain onset hours after meal, often at night, wakes patient
 Nausea
 Vomiting
 Pleuritic pain
 Fever
Give a differential diagnosis
for right upper quadrant pain
 Cholecystitis, cholelithiasis,
cholangitis, hepatitis,
carcinoma of liver, pancreas
or biliary tract
 Abdominal aneurysm
 Diverticular disease
 Gastroenteritis
 Inflammatory bowel
 Mesenteric ischaemia
 Myocardial infarction
 Pancreatitis
 Pleural effusion, right lower
lobe pneumonia or tumour
 Pregnancy: Eclampsia,
Urinary tract infection
 Renal calculi
 Right colon cancer
 Small bowel obstruction
 Fractured rib(s)
 Spinal root compression
Describe the pathogenesis of
gallstone formation (cholelithiasis)
 Bile is composed mainly of water, bile salts, lecithin (phospholipid)
and cholesterol (5%)
 Most bile flows into the gallbladder through the cystic duct, while a
small amount drains directly to duodenum
 Water is removed from bile in the gallbladder (2-5cups/day)
 70% of stones are formed of cholesterol
 30% are pigmented (black or brown)
 Cholesterol is solubilised in micelles
 In imbalance between cholesterol and bile salts
 cholesterol and calcium bilirubinate sludge
 cholesterol crystal precipitation
 Predisposing factors: high cholesterol, low gallbladder emptying and
absorption, high bilirubin
List the risk factors for
developing gallstones
 4 F’s
 Fair
 Fat
 Female
 Fertile
 Oral contraceptives or oestrogen replacement
 Pregnancy
 incidence 5.1% second trimester, 7.9% third, 10.2% 4-
6weeks postpartum
 Older age
Describe investigations that can
be used to confirm the diagnosis
of gallstones
 Ultrasound – 98% sensitive and specific
 Hepatobiliary scintigraphy (HBS) if unclear US
 Adjunctive plain radiography, CT, ERCP
 US findings for cholecystitis include
 Gallstones or sludge
 Gallbladder wall thickening (>2-4 mm)
 Gallbladder distention (diameter > 4 cm, length >10
cm)
 Pericholecystic fluid from perforation or exudate
 Air in the gallbladder wall (indicating gangrenous
cholecystitis)
 Ultrasonographic Murphy sign (86-92% sensitive, 35%
specific) - pain when the probe is pushed directly on
the gallbladder (not related to breathing)
The ultrasound shows gallstones within the gallbladder without
evidence of Cholecystitis.
 Regarding the diagnosis of cholelithiasis, which of the
following is true?
A. Offer liver function tests and ultrasound to people with
suspected gallstone disease, and to people with abdominal or
gastrointestinal symptoms that have been unresponsive to
previous management.
B. Consider endoscopic retrograde cholangio-pancreatography
(ERCP) if ultrasound has not detected common bile duct
stones but the: bile duct is dilated and/or liver function test
results are abnormal.
C. Consider T2 weighted computed tomography (CT) abdomen
with enteric contrast if MRCP does not allow a diagnosis to be
made.
D. Do not refer a person for further investigations if faecal
impaction or duodenal atresia is suspected.
 Regarding the diagnosis of cholelithiasis, which of the
following is true?
A. Offer liver function tests and ultrasound to people with
suspected gallstone disease, and to people with abdominal
or gastrointestinal symptoms that have been unresponsive
to previous management.
B. Consider endoscopic retrograde cholangio-pancreatography
(ERCP) if ultrasound has not detected common bile duct
stones but the: bile duct is dilated and/or liver function test
results are abnormal.
C. Consider T2 weighted computed tomography (CT) abdomen
with enteric contrast if MRCP does not allow a diagnosis to be
made.
D. Do not refer a person for further investigations if the cause is
suspected to be faecal impaction or duodenal atresia.
 The truth from NICE!
1. Offer liver function tests and ultrasound to people with
suspected gallstone disease, and to people with
abdominal or gastrointestinal symptoms that have been
unresponsive to previous management.
2. Consider MRCP if ultrasound has not detected common
bile duct stones but the: bile duct is dilated and/or liver
function test results are abnormal.
3. Consider endoscopic ultrasound (EUS) if MRCP does not
allow a diagnosis to be made.
4. Refer people for further investigations if conditions other
than gallstone disease are suspected.
Describe the potential
complications of gallstones
 Cholecystitis
 Gallbladder gangrene in cholecystitis (diabetics, elderly or
immunocompromised)
 In cholecystitis and/or biliary colic may be cholangitis,
sepsis, pancreatitis, hepatitis, and choledocholithiasis
 Gallbladder perforation occurs in 10% of cholecystitis
 Abscess formation
 Free perforation, bile and inflammatory release intra-
peritoneally  peritonitis
 Gallbladder enteric fistula if perforation occurs next to a
hollow organ. Most commonly duodenum.
 Gallstones may pass through the fistula into small bowel, and
if they >2.5cm can obstruct the ileocecal valve, causing
gallstone ileus. Mortality ~20%
Discuss the management of a patient
with gallstones, including the
indications for cholecystectomy
 Expectant management (watch and wait)
 Asymptomatic patients, 5mm < stones < 3cm
 Age for future surgery likelihood
 30% at 30, 20% at 50, 15% at 70
 Symptomatic antibiotics and NSAIDs (or stronger)
 Nonsurgical lithic removal
 Lithotripsy <2cm, few
 Medication: ursodiol, chenodiol <1.5cm (or contact MTBE)
 ERCP removal (endoscopic sphincterotomy)
 Surgical cholecystectomy
 Laparoscopic or open
 No gallstone protection from lowering cholesterol
 A 25 year old Hispanic woman, who is 4 months post-partum has
pain in the right upper quadrant/ Her laboratory values are as
follows: leukocyte count 9.0x103/mm3; total serum bilirubin
0.9mg/dL; serum alkaline phosphatase 100U/L; serum amylase
300 U/L. Which of the following statements regarding her
condition is INCORRECT?
A. If she undergoes a laparoscopic cholecystectomy, there is an
approximately 5% chance that conversion to an open
cholecystectomy will be necesasary
B. If she undergoes laparoscopic cholecystectomy, she will have a 1.0%
to 1.5% chance of having a surgically attributable injury to the
common bile duct
C. She is likely to have gallstones, which are common in Hispanic
women and frequently discovered during or following pregnancy
D. She most likely has experienced biliary colic and has passed a
gallstone that resulted in mild pancreatitis, rather than having
biliary obstruction at this time
E. Unless she has cholecystitis, cholelithiasis with symptomatic biliary
colic is unlikely
 A 25 year old Hispanic woman, who is 4 months post-partum has
pain in the right upper quadrant/ Her laboratory values are as
follows: leukocyte count 9.0x103/mm3; total serum bilirubin
0.9mg/dL; serum alkaline phosphatase 100U/L; serum amylase
300 U/L. Which of the following statements regarding her
condition is INCORRECT?
A. If she undergoes a laparoscopic cholecystectomy, there is an
approximately 5% chance that conversion to an open
cholecystectomy will be necesasary
B. If she undergoes laparoscopic cholecystectomy, she will have a 1.0%
to 1.5% chance of having a surgically attributable injury to the
common bile duct
C. She is likely to have gallstones, which are common in Hispanic
women and frequently discovered during or following pregnancy
D. She most likely has experienced biliary colic and has passed a
gallstone that resulted in mild pancreatitis, rather than having
biliary obstruction at this time
E. Unless she has cholecystitis, cholelithiasis with symptomatic
biliary colic is unlikely
 E. Unless she has cholecystitis, cholelithiasis with symptomatic
biliary colic is unlikely. Symptomatic and asymptomatic
cholelithiasis commonly occur in the absence of cholecystitis, an
infection of the gallbladder. Cholecystitis can develop from
bacteria alone or as a consequence of cystic duct obstruction by a
stone. Hispanic women have a high prevalence of biliary disease,
including cholelithiasis, which often occurs during and after
pregnancy. The laparoscopic approach has become the standard
technique for cholecystectomy. The frequency of common bile duct
injury is reported to range from 1.0% to 1.5%, or approximately
twice that of the open procedure. The reported need to convert to
an open cholecystectomy is 5%; to proceed safely, the surgeon
should not hesitate to convert to an open technique. The patient is
unlikely to have a bile duct obstruction, given her normal serum
bilirubin and alkaline phosphatase levels. The elevated serum
amylase level suggests that she has experienced biliary colic while
passing a stone that resulted in mild pancreatitis.
Explain to a patient the function of the
gallbladder, how, if at all, its removal may affect
them and what steps they can take to minimise
such effects
 The gallbladder is a pear-shaped organ that rests
beneath the right side of the liver.
 Its main purpose is to collect and concentrate a
digestive liquid (bile) produced by the liver. Bile is
released from the gallbladder after eating, aiding
digestion. Bile moves through bile ducts into the gut.
 Removal of the gallbladder is not associated with any
impairment of digestion in most people, but may cause
loose stools.
 Gallbladder removal is a major operation with
postoperative pain, nausea and vomiting up to 2 days.
 Recovery is within 7 days, with follow up in 2-3 weeks.
Explain to a patient the function of the
gallbladder, how, if at all, its removal may
affect them and what steps they can take to
minimise such effects
 Cholecystectomy can relieve the pain and discomfort of
gallstones
 Cholecystectomy is the only way to prevent gallstones
 Contact surgeon if:
 Persistent pain, worsening pain
 Fever >38.3C
 Continuous vomiting
 Swelling, redness, bleeding or foul smelling site
 No vowel movements 2-3 days post-op
 Increasing fibre intake should minimise digestive issues
 Balancing to a healthy weight may help
Discuss the advantages and
disadvantages of laparoscopic
surgery
 5-7 inch incision versus 4 small openings in the abdomen
 Patients usually have minimal post-operative pain
 Patients usually experience faster recovery than open
gallbladder surgery patients
 Most go home same and quicker return to normal
 Severe COPD or CHF may not tolerate CO2
pneumoperitoneum
 Gallbladder cancer is only operable by open
 Common bile duct injury (0.24%)
 Regarding laparoscopic cholecystectomy, which of the
following is correct?
A. Primarily done for cholecystitis in third trimester of
pregnancy
B. Associated with higher rate of bile duct injury than open
cholecystectomy
C. Contraindicated in acute cholecystitis
D. Safer than open cholecystectomy in patients with
cardiorespiratory disease
 Regarding laparoscopic cholecystectomy, which of the
following is correct?
A. Primarily done for cholecystitis in third trimester of
pregnancy
B. Associated with higher rate of bile duct injury than
open cholecystectomy
C. Contraindicated in acute cholecystitis
D. Safer than open cholecystectomy in patients with
cardiorespiratory disease
References
 MedLine Plus. Updated by: Laura J. Martin, MD, MPH, ABIM Board Certified in Internal
Medicine and Hospice and Palliative Medicine, Atlanta, GA. Also reviewed by David Zieve,
MD, MHA, Isla Ogilvie, PhD, and the A.D.A.M. Editorial team. Accessed Aug 14 2015.
Available from: http://www.nlm.nih.gov/medlineplus/ency/article/002237.htm
 John Hopkins Medicine Health Library. Biliary System: Anatomy and Functions Available
from:
http://www.hopkinsmedicine.org/healthlibrary/conditions/liver_biliary_and_pancreatic_dis
orders/biliary_system_anatomy_and_functions_85,P00659/
 Available from: http://www.clinbiochem.info/studentlft5.html
 Iron storage. Available from: http://library.med.utah.edu/NetBiochem/hi11b.htm
 Hyperbilirubinemia and Jaundice. University of Rochester Medical Center Health
Encyclopedia. Available from:
https://www.urmc.rochester.edu/encyclopedia/content.aspx?ContentTypeID=90&ContentID
=P02375
 NHS choices. Available from:
http://www.nhs.uk/Conditions/Jaundice/PublishingImages/M190040-
Jaundice__342x198.JPG
 Available from: http://static.guim.co.uk/sys-
images/Guardian/About/General/2010/10/20/1287571854392/Homer-Simpson-006.jpg
 Available from: http://www.ncbi.nlm.nih.gov/books/NBK413/
References
 Acute Cholecystitis and Biliary Colic. Medscape. Author: Peter A D Steel, MA, MBBS; Chief Editor:
Barry E Brenner. Accessed Aug 15 2015. Available from:
http://emedicine.medscape.com/article/1950020-overview
 http://www.uphs.upenn.edu/surgery/Education/medical_students/links/Jaundice.pdf
 http://www.aldersonfuneralhomes.com/tribute-images/666/High/Dice-2C_Joan_T-.jpg
 http://www.nlm.nih.gov/medlineplus/ency/article/000226.htm
 http://www.medscape.com/viewarticle/710045_3
 http://www.cpmc.org/advanced/liver/patients/topics/bileduct-profile.html
 http://umm.edu/health/medical/reports/articles/gallstones-and-gallbladder-disease
 http://www.sages.org/publications/patient-information/patient-information-for-laparoscopic-
gallbladder-removal-cholecystectomy-from-sages/
 http://www.mayoclinic.org/tests-procedures/cholecystectomy/basics/what-you-can-expect/prc-
20013253
 https://www.facs.org/~/media/files/education/patient%20ed/cholesys.ashx
 https://books.google.co.il/books?id=_1j3LnyNAiIC&printsec=frontcover&source=gbs_ge_summary_r&
cad=0#v=onepage&q=6.33&f=false
 http://www.netmedicos.com/?surgery/mcqs_3/page/4/
 http://www.cirse.org/files/files/EBIR/MCQ/EBIR_MCQ.pdf
 http://www.doctorsintraining.com/mkt/qa/
 http://www.nice.org.uk/guidance/cg188/resources/guidance-gallstone-disease-pdf
 When performing biliary drainage and stenting as a
palliative treatment for malignant obstructive jaundice.
Which kind of equipment is preferable?
A. Plastic retrievable endoprostheses
B. Self-expanding metallic stents
C. Balloon-expanding metallic stents
D. Covered self-expanding stents
 When performing biliary drainage and stenting as a
palliative treatment for malignant obstructive jaundice.
Which kind of equipment is preferable?
A. Plastic retrievable endoprostheses
B. Self-expanding metallic stents
C. Balloon-expanding metallic stents
D. Covered self-expanding stents

More Related Content

What's hot (20)

Hypernatremia
HypernatremiaHypernatremia
Hypernatremia
 
Approach to jaundice
Approach to jaundiceApproach to jaundice
Approach to jaundice
 
Biliary tract
Biliary tractBiliary tract
Biliary tract
 
renal function tests by Dr siva kumar
renal function tests by Dr siva kumarrenal function tests by Dr siva kumar
renal function tests by Dr siva kumar
 
Secretions of small intestine
Secretions of small intestineSecretions of small intestine
Secretions of small intestine
 
Anatomy and physiology of biliary tree
Anatomy and physiology of biliary treeAnatomy and physiology of biliary tree
Anatomy and physiology of biliary tree
 
All about Jaundice
All about JaundiceAll about Jaundice
All about Jaundice
 
Nephrolithiasis
NephrolithiasisNephrolithiasis
Nephrolithiasis
 
Bile
BileBile
Bile
 
Heme Degradation and Jaundice
Heme Degradation and JaundiceHeme Degradation and Jaundice
Heme Degradation and Jaundice
 
Bilirubin 1
Bilirubin 1Bilirubin 1
Bilirubin 1
 
Renal failure acute and chronic
Renal failure   acute and chronicRenal failure   acute and chronic
Renal failure acute and chronic
 
Chronic cholecystitis & Jaundice
Chronic cholecystitis & JaundiceChronic cholecystitis & Jaundice
Chronic cholecystitis & Jaundice
 
Liver function tests
Liver function tests Liver function tests
Liver function tests
 
Liver physiology
Liver physiologyLiver physiology
Liver physiology
 
Bilirubin metabolism
Bilirubin metabolismBilirubin metabolism
Bilirubin metabolism
 
Jaundice
JaundiceJaundice
Jaundice
 
Diagnostic tests for GI disorders
Diagnostic tests for GI disordersDiagnostic tests for GI disorders
Diagnostic tests for GI disorders
 
Liver function tests Dr.r.mallika
Liver function tests  Dr.r.mallikaLiver function tests  Dr.r.mallika
Liver function tests Dr.r.mallika
 
Hypocalcemia
Hypocalcemia Hypocalcemia
Hypocalcemia
 

Viewers also liked

Antibiotic prophylaxis in sever Acute Panceriatitis
Antibiotic prophylaxis in sever Acute PanceriatitisAntibiotic prophylaxis in sever Acute Panceriatitis
Antibiotic prophylaxis in sever Acute PanceriatitisSaeed Al-Shomimi
 
Approach patient with juandice
Approach patient with juandiceApproach patient with juandice
Approach patient with juandiceYahyia Al-abri
 
Audit of Appropriateness for Brain Scan Use for Paediatric Headache at the Em...
Audit of Appropriateness for Brain Scan Use for Paediatric Headache at the Em...Audit of Appropriateness for Brain Scan Use for Paediatric Headache at the Em...
Audit of Appropriateness for Brain Scan Use for Paediatric Headache at the Em...Lyndon Woytuck
 
Top-down gain control: Presentation on a paper by Winkowski and Knudsen
Top-down gain control: Presentation on a paper by Winkowski and KnudsenTop-down gain control: Presentation on a paper by Winkowski and Knudsen
Top-down gain control: Presentation on a paper by Winkowski and KnudsenLyndon Woytuck
 
Dynamic assignment of geospatial-temporal macro tasks to agents under human s...
Dynamic assignment of geospatial-temporal macro tasks to agents under human s...Dynamic assignment of geospatial-temporal macro tasks to agents under human s...
Dynamic assignment of geospatial-temporal macro tasks to agents under human s...Reza Nourjou, Ph.D.
 
Metodología del diseño curricular
Metodología del diseño curricularMetodología del diseño curricular
Metodología del diseño curricularRogelio Hernández
 
Mobile GIS to Form Urban Search and Rescue Teams
Mobile GIS to Form Urban Search and Rescue TeamsMobile GIS to Form Urban Search and Rescue Teams
Mobile GIS to Form Urban Search and Rescue TeamsReza Nourjou, Ph.D.
 
Integrated communication
Integrated communication Integrated communication
Integrated communication Scott Bohlin
 
Fatty acid oxidation ( Beta , Alpha omega and peroxisomal)
Fatty acid oxidation ( Beta , Alpha omega and peroxisomal)Fatty acid oxidation ( Beta , Alpha omega and peroxisomal)
Fatty acid oxidation ( Beta , Alpha omega and peroxisomal)Anup Shamsher Budhathoki
 
Frailty in the elderly
Frailty in the elderlyFrailty in the elderly
Frailty in the elderlyLyndon Woytuck
 
Supervised injection sites in Alberta
Supervised injection sites in AlbertaSupervised injection sites in Alberta
Supervised injection sites in AlbertaLyndon Woytuck
 

Viewers also liked (19)

Urine analysis
Urine analysisUrine analysis
Urine analysis
 
Jaundice
JaundiceJaundice
Jaundice
 
Antibiotic prophylaxis in sever Acute Panceriatitis
Antibiotic prophylaxis in sever Acute PanceriatitisAntibiotic prophylaxis in sever Acute Panceriatitis
Antibiotic prophylaxis in sever Acute Panceriatitis
 
Approach patient with juandice
Approach patient with juandiceApproach patient with juandice
Approach patient with juandice
 
Audit of Appropriateness for Brain Scan Use for Paediatric Headache at the Em...
Audit of Appropriateness for Brain Scan Use for Paediatric Headache at the Em...Audit of Appropriateness for Brain Scan Use for Paediatric Headache at the Em...
Audit of Appropriateness for Brain Scan Use for Paediatric Headache at the Em...
 
Nervous system
Nervous systemNervous system
Nervous system
 
Top-down gain control: Presentation on a paper by Winkowski and Knudsen
Top-down gain control: Presentation on a paper by Winkowski and KnudsenTop-down gain control: Presentation on a paper by Winkowski and Knudsen
Top-down gain control: Presentation on a paper by Winkowski and Knudsen
 
Dynamic assignment of geospatial-temporal macro tasks to agents under human s...
Dynamic assignment of geospatial-temporal macro tasks to agents under human s...Dynamic assignment of geospatial-temporal macro tasks to agents under human s...
Dynamic assignment of geospatial-temporal macro tasks to agents under human s...
 
Metodología del diseño curricular
Metodología del diseño curricularMetodología del diseño curricular
Metodología del diseño curricular
 
Mobile GIS to Form Urban Search and Rescue Teams
Mobile GIS to Form Urban Search and Rescue TeamsMobile GIS to Form Urban Search and Rescue Teams
Mobile GIS to Form Urban Search and Rescue Teams
 
Integrated communication
Integrated communication Integrated communication
Integrated communication
 
2 infancia int
2 infancia int2 infancia int
2 infancia int
 
Hypolipidaemic drugs
Hypolipidaemic drugsHypolipidaemic drugs
Hypolipidaemic drugs
 
Unit 4 endocrine system(1)
Unit 4 endocrine system(1)Unit 4 endocrine system(1)
Unit 4 endocrine system(1)
 
Fatty acid oxidation ( Beta , Alpha omega and peroxisomal)
Fatty acid oxidation ( Beta , Alpha omega and peroxisomal)Fatty acid oxidation ( Beta , Alpha omega and peroxisomal)
Fatty acid oxidation ( Beta , Alpha omega and peroxisomal)
 
Frailty in the elderly
Frailty in the elderlyFrailty in the elderly
Frailty in the elderly
 
Digestion!
Digestion!Digestion!
Digestion!
 
Statins
StatinsStatins
Statins
 
Supervised injection sites in Alberta
Supervised injection sites in AlbertaSupervised injection sites in Alberta
Supervised injection sites in Alberta
 

Similar to What is Bile and How Does it Aid Digestion

Evaluation of Jaundice by Dr. Sookun Rajeev Kumar
Evaluation of Jaundice by Dr. Sookun Rajeev KumarEvaluation of Jaundice by Dr. Sookun Rajeev Kumar
Evaluation of Jaundice by Dr. Sookun Rajeev KumarDr. Sookun Rajeev Kumar
 
HEPATIC DISORDERS .pptx
HEPATIC DISORDERS .pptxHEPATIC DISORDERS .pptx
HEPATIC DISORDERS .pptxMesfinShifara
 
jaundice-160414200805.pptx
jaundice-160414200805.pptxjaundice-160414200805.pptx
jaundice-160414200805.pptxSushma263211
 
Approach to cholestatic jaundice
Approach to cholestatic jaundiceApproach to cholestatic jaundice
Approach to cholestatic jaundiceRam Raut
 
Presentation Mdc Nephro Uro
Presentation Mdc Nephro UroPresentation Mdc Nephro Uro
Presentation Mdc Nephro UroMiami Dade
 
Nutrition and liver diseases by Dr.P.Nazni
Nutrition and liver diseases by Dr.P.NazniNutrition and liver diseases by Dr.P.Nazni
Nutrition and liver diseases by Dr.P.Nazninazni peerkhan
 
Hepatic disorder ! Cirrhosis, Jaundice
Hepatic disorder ! Cirrhosis, Jaundice Hepatic disorder ! Cirrhosis, Jaundice
Hepatic disorder ! Cirrhosis, Jaundice Rahul Ranjan
 
Hepatic disorders
Hepatic disordersHepatic disorders
Hepatic disordersDAWN V TOMY
 
Symptoms Of Gastrointestinal Tract and its causes. Part 2
Symptoms Of Gastrointestinal Tract and its causes. Part 2Symptoms Of Gastrointestinal Tract and its causes. Part 2
Symptoms Of Gastrointestinal Tract and its causes. Part 2Ahsan Sajjad
 
Approach To A Patient With Jaundice
Approach To A Patient With JaundiceApproach To A Patient With Jaundice
Approach To A Patient With JaundiceTanuj Bhatia
 
Chronic diarrhoea
Chronic diarrhoeaChronic diarrhoea
Chronic diarrhoeaVarun Karri
 
Chronic diarrhoea
Chronic diarrhoeaChronic diarrhoea
Chronic diarrhoeaVarun Karri
 

Similar to What is Bile and How Does it Aid Digestion (20)

Liver disease
Liver diseaseLiver disease
Liver disease
 
Evaluation of Jaundice by Dr. Sookun Rajeev Kumar
Evaluation of Jaundice by Dr. Sookun Rajeev KumarEvaluation of Jaundice by Dr. Sookun Rajeev Kumar
Evaluation of Jaundice by Dr. Sookun Rajeev Kumar
 
HEPATIC DISORDERS .pptx
HEPATIC DISORDERS .pptxHEPATIC DISORDERS .pptx
HEPATIC DISORDERS .pptx
 
The liver
The liverThe liver
The liver
 
jaundice-160414200805.pptx
jaundice-160414200805.pptxjaundice-160414200805.pptx
jaundice-160414200805.pptx
 
Approach to cholestatic jaundice
Approach to cholestatic jaundiceApproach to cholestatic jaundice
Approach to cholestatic jaundice
 
Hepatic disease
Hepatic diseaseHepatic disease
Hepatic disease
 
evaluation of jaundice
evaluation of jaundiceevaluation of jaundice
evaluation of jaundice
 
Hepatitis C
Hepatitis CHepatitis C
Hepatitis C
 
Presentation Mdc Nephro Uro
Presentation Mdc Nephro UroPresentation Mdc Nephro Uro
Presentation Mdc Nephro Uro
 
Nutrition and liver diseases by Dr.P.Nazni
Nutrition and liver diseases by Dr.P.NazniNutrition and liver diseases by Dr.P.Nazni
Nutrition and liver diseases by Dr.P.Nazni
 
Hepatic disorder ! Cirrhosis, Jaundice
Hepatic disorder ! Cirrhosis, Jaundice Hepatic disorder ! Cirrhosis, Jaundice
Hepatic disorder ! Cirrhosis, Jaundice
 
Jaundice
JaundiceJaundice
Jaundice
 
Direct jaundice
Direct jaundiceDirect jaundice
Direct jaundice
 
Hepatic disorders
Hepatic disordersHepatic disorders
Hepatic disorders
 
Jaundice
JaundiceJaundice
Jaundice
 
Symptoms Of Gastrointestinal Tract and its causes. Part 2
Symptoms Of Gastrointestinal Tract and its causes. Part 2Symptoms Of Gastrointestinal Tract and its causes. Part 2
Symptoms Of Gastrointestinal Tract and its causes. Part 2
 
Approach To A Patient With Jaundice
Approach To A Patient With JaundiceApproach To A Patient With Jaundice
Approach To A Patient With Jaundice
 
Chronic diarrhoea
Chronic diarrhoeaChronic diarrhoea
Chronic diarrhoea
 
Chronic diarrhoea
Chronic diarrhoeaChronic diarrhoea
Chronic diarrhoea
 

More from Lyndon Woytuck

A case study in early pregnancy loss
A case study in early pregnancy lossA case study in early pregnancy loss
A case study in early pregnancy lossLyndon Woytuck
 
Diabetic ketoacidosis in pregnancy
Diabetic ketoacidosis in pregnancyDiabetic ketoacidosis in pregnancy
Diabetic ketoacidosis in pregnancyLyndon Woytuck
 
Mental illness in a societal context
Mental illness in a societal contextMental illness in a societal context
Mental illness in a societal contextLyndon Woytuck
 
Poster: Audit of Appropriateness for Brain Scan Use for Paediatric Headache a...
Poster: Audit of Appropriateness for Brain Scan Use for Paediatric Headache a...Poster: Audit of Appropriateness for Brain Scan Use for Paediatric Headache a...
Poster: Audit of Appropriateness for Brain Scan Use for Paediatric Headache a...Lyndon Woytuck
 
Mesenteric ischaemia: a review
Mesenteric ischaemia: a reviewMesenteric ischaemia: a review
Mesenteric ischaemia: a reviewLyndon Woytuck
 
Herpes zoster: a case and review
Herpes zoster: a case and reviewHerpes zoster: a case and review
Herpes zoster: a case and reviewLyndon Woytuck
 
Diabetic ketoacidosis: a case study
Diabetic ketoacidosis: a case studyDiabetic ketoacidosis: a case study
Diabetic ketoacidosis: a case studyLyndon Woytuck
 
Heparin induced thrombocytopaenia: HIT
Heparin induced thrombocytopaenia: HITHeparin induced thrombocytopaenia: HIT
Heparin induced thrombocytopaenia: HITLyndon Woytuck
 
Neuroblastoma: a review
Neuroblastoma: a reviewNeuroblastoma: a review
Neuroblastoma: a reviewLyndon Woytuck
 
Asthma exacerbation case study in pediatrics
Asthma exacerbation case study in pediatricsAsthma exacerbation case study in pediatrics
Asthma exacerbation case study in pediatricsLyndon Woytuck
 
Juvenile glaucoma: a case study and disease review
Juvenile glaucoma: a case study and disease reviewJuvenile glaucoma: a case study and disease review
Juvenile glaucoma: a case study and disease reviewLyndon Woytuck
 
Case Study: Recurrent myoma with menorrhagia
Case Study: Recurrent myoma with menorrhagiaCase Study: Recurrent myoma with menorrhagia
Case Study: Recurrent myoma with menorrhagiaLyndon Woytuck
 
Steroids: a summary for care
Steroids: a summary for careSteroids: a summary for care
Steroids: a summary for careLyndon Woytuck
 
Anticoagulation therapy for atrial fibrillation
Anticoagulation therapy for atrial fibrillationAnticoagulation therapy for atrial fibrillation
Anticoagulation therapy for atrial fibrillationLyndon Woytuck
 

More from Lyndon Woytuck (14)

A case study in early pregnancy loss
A case study in early pregnancy lossA case study in early pregnancy loss
A case study in early pregnancy loss
 
Diabetic ketoacidosis in pregnancy
Diabetic ketoacidosis in pregnancyDiabetic ketoacidosis in pregnancy
Diabetic ketoacidosis in pregnancy
 
Mental illness in a societal context
Mental illness in a societal contextMental illness in a societal context
Mental illness in a societal context
 
Poster: Audit of Appropriateness for Brain Scan Use for Paediatric Headache a...
Poster: Audit of Appropriateness for Brain Scan Use for Paediatric Headache a...Poster: Audit of Appropriateness for Brain Scan Use for Paediatric Headache a...
Poster: Audit of Appropriateness for Brain Scan Use for Paediatric Headache a...
 
Mesenteric ischaemia: a review
Mesenteric ischaemia: a reviewMesenteric ischaemia: a review
Mesenteric ischaemia: a review
 
Herpes zoster: a case and review
Herpes zoster: a case and reviewHerpes zoster: a case and review
Herpes zoster: a case and review
 
Diabetic ketoacidosis: a case study
Diabetic ketoacidosis: a case studyDiabetic ketoacidosis: a case study
Diabetic ketoacidosis: a case study
 
Heparin induced thrombocytopaenia: HIT
Heparin induced thrombocytopaenia: HITHeparin induced thrombocytopaenia: HIT
Heparin induced thrombocytopaenia: HIT
 
Neuroblastoma: a review
Neuroblastoma: a reviewNeuroblastoma: a review
Neuroblastoma: a review
 
Asthma exacerbation case study in pediatrics
Asthma exacerbation case study in pediatricsAsthma exacerbation case study in pediatrics
Asthma exacerbation case study in pediatrics
 
Juvenile glaucoma: a case study and disease review
Juvenile glaucoma: a case study and disease reviewJuvenile glaucoma: a case study and disease review
Juvenile glaucoma: a case study and disease review
 
Case Study: Recurrent myoma with menorrhagia
Case Study: Recurrent myoma with menorrhagiaCase Study: Recurrent myoma with menorrhagia
Case Study: Recurrent myoma with menorrhagia
 
Steroids: a summary for care
Steroids: a summary for careSteroids: a summary for care
Steroids: a summary for care
 
Anticoagulation therapy for atrial fibrillation
Anticoagulation therapy for atrial fibrillationAnticoagulation therapy for atrial fibrillation
Anticoagulation therapy for atrial fibrillation
 

Recently uploaded

Bangalore Call Girls Marathahalli 📞 9907093804 High Profile Service 100% Safe
Bangalore Call Girls Marathahalli 📞 9907093804 High Profile Service 100% SafeBangalore Call Girls Marathahalli 📞 9907093804 High Profile Service 100% Safe
Bangalore Call Girls Marathahalli 📞 9907093804 High Profile Service 100% Safenarwatsonia7
 
Call Girls Jp Nagar Just Call 7001305949 Top Class Call Girl Service Available
Call Girls Jp Nagar Just Call 7001305949 Top Class Call Girl Service AvailableCall Girls Jp Nagar Just Call 7001305949 Top Class Call Girl Service Available
Call Girls Jp Nagar Just Call 7001305949 Top Class Call Girl Service Availablenarwatsonia7
 
Russian Call Girls Chickpet - 7001305949 Booking and charges genuine rate for...
Russian Call Girls Chickpet - 7001305949 Booking and charges genuine rate for...Russian Call Girls Chickpet - 7001305949 Booking and charges genuine rate for...
Russian Call Girls Chickpet - 7001305949 Booking and charges genuine rate for...narwatsonia7
 
Call Girl Koramangala | 7001305949 At Low Cost Cash Payment Booking
Call Girl Koramangala | 7001305949 At Low Cost Cash Payment BookingCall Girl Koramangala | 7001305949 At Low Cost Cash Payment Booking
Call Girl Koramangala | 7001305949 At Low Cost Cash Payment Bookingnarwatsonia7
 
Call Girls Hosur Just Call 7001305949 Top Class Call Girl Service Available
Call Girls Hosur Just Call 7001305949 Top Class Call Girl Service AvailableCall Girls Hosur Just Call 7001305949 Top Class Call Girl Service Available
Call Girls Hosur Just Call 7001305949 Top Class Call Girl Service Availablenarwatsonia7
 
Call Girl Lucknow Mallika 7001305949 Independent Escort Service Lucknow
Call Girl Lucknow Mallika 7001305949 Independent Escort Service LucknowCall Girl Lucknow Mallika 7001305949 Independent Escort Service Lucknow
Call Girl Lucknow Mallika 7001305949 Independent Escort Service Lucknownarwatsonia7
 
Kolkata Call Girls Services 9907093804 @24x7 High Class Babes Here Call Now
Kolkata Call Girls Services 9907093804 @24x7 High Class Babes Here Call NowKolkata Call Girls Services 9907093804 @24x7 High Class Babes Here Call Now
Kolkata Call Girls Services 9907093804 @24x7 High Class Babes Here Call NowNehru place Escorts
 
Housewife Call Girls Bangalore - Call 7001305949 Rs-3500 with A/C Room Cash o...
Housewife Call Girls Bangalore - Call 7001305949 Rs-3500 with A/C Room Cash o...Housewife Call Girls Bangalore - Call 7001305949 Rs-3500 with A/C Room Cash o...
Housewife Call Girls Bangalore - Call 7001305949 Rs-3500 with A/C Room Cash o...narwatsonia7
 
Call Girls Hebbal Just Call 7001305949 Top Class Call Girl Service Available
Call Girls Hebbal Just Call 7001305949 Top Class Call Girl Service AvailableCall Girls Hebbal Just Call 7001305949 Top Class Call Girl Service Available
Call Girls Hebbal Just Call 7001305949 Top Class Call Girl Service Availablenarwatsonia7
 
Call Girl Bangalore Nandini 7001305949 Independent Escort Service Bangalore
Call Girl Bangalore Nandini 7001305949 Independent Escort Service BangaloreCall Girl Bangalore Nandini 7001305949 Independent Escort Service Bangalore
Call Girl Bangalore Nandini 7001305949 Independent Escort Service Bangalorenarwatsonia7
 
College Call Girls Vyasarpadi Whatsapp 7001305949 Independent Escort Service
College Call Girls Vyasarpadi Whatsapp 7001305949 Independent Escort ServiceCollege Call Girls Vyasarpadi Whatsapp 7001305949 Independent Escort Service
College Call Girls Vyasarpadi Whatsapp 7001305949 Independent Escort ServiceNehru place Escorts
 
Book Call Girls in Kasavanahalli - 7001305949 with real photos and phone numbers
Book Call Girls in Kasavanahalli - 7001305949 with real photos and phone numbersBook Call Girls in Kasavanahalli - 7001305949 with real photos and phone numbers
Book Call Girls in Kasavanahalli - 7001305949 with real photos and phone numbersnarwatsonia7
 
VIP Call Girls Lucknow Nandini 7001305949 Independent Escort Service Lucknow
VIP Call Girls Lucknow Nandini 7001305949 Independent Escort Service LucknowVIP Call Girls Lucknow Nandini 7001305949 Independent Escort Service Lucknow
VIP Call Girls Lucknow Nandini 7001305949 Independent Escort Service Lucknownarwatsonia7
 
Bangalore Call Girls Majestic 📞 9907093804 High Profile Service 100% Safe
Bangalore Call Girls Majestic 📞 9907093804 High Profile Service 100% SafeBangalore Call Girls Majestic 📞 9907093804 High Profile Service 100% Safe
Bangalore Call Girls Majestic 📞 9907093804 High Profile Service 100% Safenarwatsonia7
 
Call Girls ITPL Just Call 7001305949 Top Class Call Girl Service Available
Call Girls ITPL Just Call 7001305949 Top Class Call Girl Service AvailableCall Girls ITPL Just Call 7001305949 Top Class Call Girl Service Available
Call Girls ITPL Just Call 7001305949 Top Class Call Girl Service Availablenarwatsonia7
 
Call Girls Thane Just Call 9910780858 Get High Class Call Girls Service
Call Girls Thane Just Call 9910780858 Get High Class Call Girls ServiceCall Girls Thane Just Call 9910780858 Get High Class Call Girls Service
Call Girls Thane Just Call 9910780858 Get High Class Call Girls Servicesonalikaur4
 
Call Girls Budhwar Peth 7001305949 All Area Service COD available Any Time
Call Girls Budhwar Peth 7001305949 All Area Service COD available Any TimeCall Girls Budhwar Peth 7001305949 All Area Service COD available Any Time
Call Girls Budhwar Peth 7001305949 All Area Service COD available Any Timevijaych2041
 
Call Girls Jayanagar Just Call 7001305949 Top Class Call Girl Service Available
Call Girls Jayanagar Just Call 7001305949 Top Class Call Girl Service AvailableCall Girls Jayanagar Just Call 7001305949 Top Class Call Girl Service Available
Call Girls Jayanagar Just Call 7001305949 Top Class Call Girl Service Availablenarwatsonia7
 

Recently uploaded (20)

Bangalore Call Girls Marathahalli 📞 9907093804 High Profile Service 100% Safe
Bangalore Call Girls Marathahalli 📞 9907093804 High Profile Service 100% SafeBangalore Call Girls Marathahalli 📞 9907093804 High Profile Service 100% Safe
Bangalore Call Girls Marathahalli 📞 9907093804 High Profile Service 100% Safe
 
Call Girls Jp Nagar Just Call 7001305949 Top Class Call Girl Service Available
Call Girls Jp Nagar Just Call 7001305949 Top Class Call Girl Service AvailableCall Girls Jp Nagar Just Call 7001305949 Top Class Call Girl Service Available
Call Girls Jp Nagar Just Call 7001305949 Top Class Call Girl Service Available
 
Russian Call Girls Chickpet - 7001305949 Booking and charges genuine rate for...
Russian Call Girls Chickpet - 7001305949 Booking and charges genuine rate for...Russian Call Girls Chickpet - 7001305949 Booking and charges genuine rate for...
Russian Call Girls Chickpet - 7001305949 Booking and charges genuine rate for...
 
Call Girl Koramangala | 7001305949 At Low Cost Cash Payment Booking
Call Girl Koramangala | 7001305949 At Low Cost Cash Payment BookingCall Girl Koramangala | 7001305949 At Low Cost Cash Payment Booking
Call Girl Koramangala | 7001305949 At Low Cost Cash Payment Booking
 
Call Girls Hosur Just Call 7001305949 Top Class Call Girl Service Available
Call Girls Hosur Just Call 7001305949 Top Class Call Girl Service AvailableCall Girls Hosur Just Call 7001305949 Top Class Call Girl Service Available
Call Girls Hosur Just Call 7001305949 Top Class Call Girl Service Available
 
Call Girl Lucknow Mallika 7001305949 Independent Escort Service Lucknow
Call Girl Lucknow Mallika 7001305949 Independent Escort Service LucknowCall Girl Lucknow Mallika 7001305949 Independent Escort Service Lucknow
Call Girl Lucknow Mallika 7001305949 Independent Escort Service Lucknow
 
Kolkata Call Girls Services 9907093804 @24x7 High Class Babes Here Call Now
Kolkata Call Girls Services 9907093804 @24x7 High Class Babes Here Call NowKolkata Call Girls Services 9907093804 @24x7 High Class Babes Here Call Now
Kolkata Call Girls Services 9907093804 @24x7 High Class Babes Here Call Now
 
Housewife Call Girls Bangalore - Call 7001305949 Rs-3500 with A/C Room Cash o...
Housewife Call Girls Bangalore - Call 7001305949 Rs-3500 with A/C Room Cash o...Housewife Call Girls Bangalore - Call 7001305949 Rs-3500 with A/C Room Cash o...
Housewife Call Girls Bangalore - Call 7001305949 Rs-3500 with A/C Room Cash o...
 
Call Girls Hebbal Just Call 7001305949 Top Class Call Girl Service Available
Call Girls Hebbal Just Call 7001305949 Top Class Call Girl Service AvailableCall Girls Hebbal Just Call 7001305949 Top Class Call Girl Service Available
Call Girls Hebbal Just Call 7001305949 Top Class Call Girl Service Available
 
Call Girl Bangalore Nandini 7001305949 Independent Escort Service Bangalore
Call Girl Bangalore Nandini 7001305949 Independent Escort Service BangaloreCall Girl Bangalore Nandini 7001305949 Independent Escort Service Bangalore
Call Girl Bangalore Nandini 7001305949 Independent Escort Service Bangalore
 
College Call Girls Vyasarpadi Whatsapp 7001305949 Independent Escort Service
College Call Girls Vyasarpadi Whatsapp 7001305949 Independent Escort ServiceCollege Call Girls Vyasarpadi Whatsapp 7001305949 Independent Escort Service
College Call Girls Vyasarpadi Whatsapp 7001305949 Independent Escort Service
 
Book Call Girls in Kasavanahalli - 7001305949 with real photos and phone numbers
Book Call Girls in Kasavanahalli - 7001305949 with real photos and phone numbersBook Call Girls in Kasavanahalli - 7001305949 with real photos and phone numbers
Book Call Girls in Kasavanahalli - 7001305949 with real photos and phone numbers
 
VIP Call Girls Lucknow Nandini 7001305949 Independent Escort Service Lucknow
VIP Call Girls Lucknow Nandini 7001305949 Independent Escort Service LucknowVIP Call Girls Lucknow Nandini 7001305949 Independent Escort Service Lucknow
VIP Call Girls Lucknow Nandini 7001305949 Independent Escort Service Lucknow
 
Bangalore Call Girls Majestic 📞 9907093804 High Profile Service 100% Safe
Bangalore Call Girls Majestic 📞 9907093804 High Profile Service 100% SafeBangalore Call Girls Majestic 📞 9907093804 High Profile Service 100% Safe
Bangalore Call Girls Majestic 📞 9907093804 High Profile Service 100% Safe
 
Call Girls ITPL Just Call 7001305949 Top Class Call Girl Service Available
Call Girls ITPL Just Call 7001305949 Top Class Call Girl Service AvailableCall Girls ITPL Just Call 7001305949 Top Class Call Girl Service Available
Call Girls ITPL Just Call 7001305949 Top Class Call Girl Service Available
 
Call Girls Thane Just Call 9910780858 Get High Class Call Girls Service
Call Girls Thane Just Call 9910780858 Get High Class Call Girls ServiceCall Girls Thane Just Call 9910780858 Get High Class Call Girls Service
Call Girls Thane Just Call 9910780858 Get High Class Call Girls Service
 
Russian Call Girls in Delhi Tanvi ➡️ 9711199012 💋📞 Independent Escort Service...
Russian Call Girls in Delhi Tanvi ➡️ 9711199012 💋📞 Independent Escort Service...Russian Call Girls in Delhi Tanvi ➡️ 9711199012 💋📞 Independent Escort Service...
Russian Call Girls in Delhi Tanvi ➡️ 9711199012 💋📞 Independent Escort Service...
 
Call Girls Budhwar Peth 7001305949 All Area Service COD available Any Time
Call Girls Budhwar Peth 7001305949 All Area Service COD available Any TimeCall Girls Budhwar Peth 7001305949 All Area Service COD available Any Time
Call Girls Budhwar Peth 7001305949 All Area Service COD available Any Time
 
sauth delhi call girls in Bhajanpura 🔝 9953056974 🔝 escort Service
sauth delhi call girls in Bhajanpura 🔝 9953056974 🔝 escort Servicesauth delhi call girls in Bhajanpura 🔝 9953056974 🔝 escort Service
sauth delhi call girls in Bhajanpura 🔝 9953056974 🔝 escort Service
 
Call Girls Jayanagar Just Call 7001305949 Top Class Call Girl Service Available
Call Girls Jayanagar Just Call 7001305949 Top Class Call Girl Service AvailableCall Girls Jayanagar Just Call 7001305949 Top Class Call Girl Service Available
Call Girls Jayanagar Just Call 7001305949 Top Class Call Girl Service Available
 

What is Bile and How Does it Aid Digestion

  • 2. What is bile?  Bile is a fluid made by liver cells (hepatocytes), secreted into the biliary tract and stored in the gallbladder.  Bile aids in digestion by breaking down fats into fatty acids, which can be absorbed by the digestive tract for further use by the body.  Bile is mainly composed of cholesterol, bile acids (or bile salts), and bilirubin (a breakdown product of red blood cells). Bile also contains water, potassium and sodium salts, copper and trace metals
  • 3. A note on the Anatomy  The common hepatic duct then merges with the cystic duct from the gallbladder to form the common bile duct.  The common bile duct joins with the duodenum at the Ampulla of Vater.  About half of the bile produced by the liver is first stored in the gallbladder before emptying into the duodenum through the sphincter of Oddi.  The gallbladder is a pear-shaped organ attached beneath the liver.  When food enters the duodenum, the gallbladder contracts and releases stored bile into the intestinal lumen to aid digestion.  Bile secreted from hepatocytes is collected by a system of ducts flowing within the liver and eventually draining into the left and right hepatic ducts.  The hepatic ducts drain into the common hepatic duct.
  • 4. Naturally, the physiology  The biliary system drains waste products from the liver into the duodenum,  And aids in digestion via CCK controlled release of bile  Bile is a greenish-yellow fluid and serves two primary functions:  Excretion of waste  Fat digestion  Bile acids emulsify and absorb fats with vitamins A, D, E, K  Stercobilin is excreted from the body in faeces, and is what gives faeces its dark brown color.  Urobilin is carried through the blood plasma and excreted in urine, giving urine a yellow (sometimes brown) colour
  • 5. For the image shown, answer the following questions: 1. Identify the labeled structures (A-G). 2. What is the name of the sphincter that controls the structure labeled (L)? 3. What life-threatening complication can develop as a result of a gallstone becoming lodged within the structure labeled (L)? 4. BONUS: Which hormone stimulates the release of bile from the gallbladder? Answer: 1. (A) Right hepatic duct, (B) left hepatic duct, (C) cystic duct, (D) common hepatic duct, (E) common bile duct, (F) accessory pancreatic duct, and (G) pancreatic duct. 2. The sphincter responsible for controlling the major duodenal papilla (ampulla of Vater) (L) is the sphincter of Oddi. 3. Gallstone-induced pancreatitis can result from a gallstone that becomes lodged within the major duodenal papilla (ampulla of Vater) (L). 4. Cholecystokinin (CCK), which is produced by I cells of the duodenum and jejunum, stimulates the gallbladder to contract and release bile.
  • 6. Explain how disruption of bilirubin metabolism and excretion can cause jaundice  Jaundice is caused by an accumulation of bile pigments (biliverdin, bilirubin) in the body. This is usually first visible in mucus membranes (sclerae), then followed by the skin.  Haemolytic  increased haemoglobin breakdown produces more bilirubin, which overloads the conjugation. Increase in unconjugated form. Other LFTs usually normal, with reduced haptoglobin, reticulocytosis and abnormal red cells on film. Anaemia may occur.  Hepatocellular  failure of conjugation resulting in increased bilirubin. Increased ALT (marker of hepatocellular damage). Damage to hepatocytes may cause leakage of bilirubin into plasma and conjugated bilirubin may be found in the urine.  Obstructive  biliary obstruction results in conjugated bilirubin to pass from the liver to plasma. Very high levels may cause the patient to turn green and turn the urine dark. Less stercobilin and faeces turn pale. Elevated ALP due to cholestasis. Carotenaemia or quinacrine ingestion can result in yellow or green skin colour precipitated by eating large amounts of green and yellow vegetables, corn or tomatoes. In absence of yellow mucous membranes and sclerae, with normal urine color, and accentuated yellow-brown pigment on palms, soles, and nasolabial folds. Quinacrine used for treatment of giardiasis may colour skin yellow, but with normal urine. Serum bilirubin levels are normal.
  • 7. Describe the pathogenesis of conjugated and unconjugated hyperbilirubinaemia and list conditions associated with each  Conjugated:  Obstructive – cholestasis, cholangitis  Hepatocellular – infection, hepatocellular carcinoma, cirrhosis  Physiologic jaundice of the newborn  Breast milk jaundice  Dubin-Johnson's / Rotor's: impairment of hepatocyte bilirubin secretion  Unconjugated:  Haemolytic – haemolytic disease of the newborn (Rh), Wilson’s, autoimmune hepatitis, alcoholic, drug induced  Ineffective erythropoiesis  Gilbert's: hepatocyte bilirubin uptake alteration  Crigler-Najjar's: glucuronyl transferase deficiency
  • 8.  Lab report of liver function tests of a patient with jaundice showed predominantly conjugated hyperbilirubinaemia. Which of the following conditions is most likely to be responsible for conjugated hyperbilirubinaemia? A. Acute haemolytic crisis in sickle cell disease B. Gilbert syndrome C. Haemolysis due to rhesus incompatibility D. Obstructive jaundice due to carcinoma of common bile duct E. Physiological jaundice of the newborn
  • 9.  Lab report of liver function tests of a patient with jaundice showed predominantly conjugated hyperbilirubinaemia. Which of the following conditions is most likely to be responsible for conjugated hyperbilirubinaemia? A. Acute haemolytic crisis in sickle cell disease B. Gilbert syndrome C. Haemolysis due to rhesus incompatibility D. Obstructive jaundice due to carcinoma of common bile duct E. Physiological jaundice of the newborn
  • 10. Describe the symptoms and signs in a patient presenting with jaundice  Yellow colouring of the skin and eyes (usually beginning on the face and moving down the body)  Direct and indirect bilirubin levels. These reflect whether the bilirubin is bound with other substances by the liver so that it can be excreted (direct), or is circulating in the blood circulation (indirect). Either measurement may be high.  Red blood cell counts. Haemolytic jaundice may present with anaemia or reticulocytosis.
  • 11. Explain how features of the history and examination can be used to distinguish between haemolytic, hepatocellular or cholestatic jaundice  Onset  Few days, one week – hepatitis: viral, bacterial, drug or toxin  Weeks – subacute hepatitis, extrahepatic obstruction  Fluctuating intensity – cholelithiasis, ampullary cancer, drug hepatitis  Past history – chronic hepatitis, cirrhosis, benign recurrent intrahepatic cholestasis, genetic non-haemolytic hyperbilirubinaemia  Age  >40 pancreatic carcinoma, cholelithiasis  <25 Hepatitis A  Constitutional symptoms – anorexia, nausea, emesis, weight loss  within 2 weeks prior: hepatitis or cholelithiasis  Continuously >2 weeks: malignant biliary obstruction, chronic hepatitis, toxin (alcohol)  Recurrent brief episodes over months/years and RUQ pain: cholelithiasis
  • 12. Features of History and Exam: Symptoms  Abdominal pain  Dull ache RUQ: acute hepatitis  Acute abdomen (fever, jaundice and leucocytosis): alcoholic hepatitis  RUQ episodically with radiation to right scapula or girdle distribution: cholelithiasis  Epigastric or RUQ with radiation to back: pancreatic head carcinoma  Fever  Acute hepatitis or with chills, biliary obstruction (stones or stricture >> malignancy)  Pruritis  Biliary tract obstruction, occasionally viral hepatitis  Recent onset: large ducts – neoplasm, or canaliculi – intrahepatic (drug)  Long standing, middle-aged woman: primary biliary cirrhosis
  • 13. Features of History and Examination: Portals of entry  Drug and toxin exposures  Pain relievers, tranquilizers, oestrogens, chemicals, alcohol  Hepatic infection (viral, etc.)  Patients, transfusions, needles, narcotics, raw shellfish, travel (entamoeba), sexual, animals or stagnant water (leptospirosis), immunocompromise, infectious mono, sepsis  Surgery (<3 weeks)  Increased bilirubin load: transfusion haemolysis, haematoma resorption and G6PDd, drug reactions, malarial transfusion  Impaired hepatic function: halogenated anaesthesia, drugs, sepsis, hepatic ischaemia  Obstruction: surgical injury, biliary calculus, cholecystitis  Previous biliary surgery (<2 years): biliary stricture
  • 14. Features of History and Examination: Historical insights  Systemic conditions:  Inflammatory bowel disease  primary sclerosing cholangitis, cholangiocarcinoma, chronic hepatitis, cirrhosis, hepatic amyloid. Resection – cholelithiasis  Cirrhosis  Cystic fibrosis, haemochromatosis, Wilson’s, alpha-1 antitrypsin def., galactosaemia, hereditary fructose intolerance, tyrosinaemia  Systemic lupus erythematosus  Hypoxic injury – hepatitis  Family history of jaundice or hepatic disease  Genetically transmitted non-haemolytic hyperbilirubinaemias: Crigler- Najjar, Gilbert’s, Dubin-Johnson, Rotor’s  Benign recurrent intrahepatic cholestasis, Wilson’s disease, haemochromatosis, alpha-1 antitrypsin def, hereditary spherocytosis
  • 15. Features of History and Examination: Physical Inspection  Hepatocellular diseased patient appears more acutely ill than obstructive disease  Greenish jaundice – prolonged obstruction  Orange-yellow jaundice – hepatocellular  Mental derangements – hepatocellular >> obstruction  Spider telangiectasias: chronic hepatocellular  Scratch marks: pruritis  Decreased axillary/pubic hair, gynaecoid changes: cirrhosis  Dupuytren’s contracture: chronic liver disease  Xanthelasma, tuberous xanthomas: long standing biliary obstruction with hyperlipidaemia, primary biliary cirrhosis
  • 16. Features of History and Examination: Physical Auscultation, Palpation  Hepatic bruits  malignancy, hepatitis, haemangioma  Hepatic friction rub  malignancy, inflammatory (Glisson’s capsule)  Standing up may aid here  Very large liver  congested or fatty cirrhosis, neoplasm or amyloid  Rapidly shrinking liver  acute liver failure – viral or toxin  Hard or nodular  fibrotic or malignant infiltration
  • 17. Features of History and Examination: Palpation, Percussion  Unusual tenderness  acute hepatitis, abscess or rapid enlargement (vascular congestion or fatty changes)  Splenomegaly  w/out hepatomegaly: primary haemolytic or portal vein occlusion  Portal hypertension, viral hepatitis  w/ hepatomegaly: malignancy (haematologic) or storage disease  Ascites  cirrhosis with portal hypertension or malignancy >> massive or subacute hepatic necrosis or hepatic vein obstruction
  • 18.  A 52 year old man, who was previously well, presented with a 3 week history of increasing jaundice associated with pale stools, dark urine and itching. There was no associated pain or fever. He was not on any medication. Blood tests confirmed a cholestatic pattern of hyperbilirubinaemia with markedly raised alkaline phosphatase, moderate elevation of transaminases and normal serum albumin. Regarding the planning of his further investigation, which is true A. Absence of pain excludes gallstone obstruction of the common bile duct (CBD) B. Absence of bile duct dilatation on ultrasound scan makes it safe to immediately proceed to liver biopsy C. Magnetic resonance cholangio-pancreatography (MRCP) is inferior to endoscopic retrograde cholangio-pancreatography (ERCP) in the investigation of extrahepatic cholestasis where ultrasonography fails to show the cause D. The role of ERCP is primarily to undertake therapeutic measures, which might avoid surgical intervention E. A prolonged prothrombin time due to extra-hepatic cholestasis requires a 5 day course of intravenous vitamin K (5mg) to correct
  • 19.  A 52 year old man, who was previously well, presented with a 3 week history of increasing jaundice associated with pale stools, dark urine and itching. There was no associated pain or fever. He was not on any medication. Blood tests confirmed a cholestatic pattern of hyperbilirubinaemia with markedly raised alkaline phosphatase, moderate elevation of transaminases and normal serum albumin. Regarding the planning of his further investigation, which is true A. Absence of pain excludes gallstone obstruction of the common bile duct (CBD) B. Absence of bile duct dilatation on ultrasound scan makes it safe to immediately proceed to liver biopsy C. Magnetic resonance cholangio-pancreatography (MRCP) is inferior to endoscopic retrograde cholangio-pancreatography (ERCP) in the investigation of extrahepatic cholestasis where ultrasonography fails to show the cause D. The role of ERCP is primarily to undertake therapeutic measures, which might avoid surgical intervention E. A prolonged prothrombin time due to extra-hepatic cholestasis requires a 5 day course of intravenous vitamin K (5mg) to correct
  • 20.  Correct answer – D (www.medicinecpd.co.uk) Explanation: Absence of pain does not exclude the passage of gallstones obstructing the CBD; even the appearance of bile duct dilatation may not be obvious on USG if the obstruction is intermittent. In this situation MRCP should be undertaken and the more invasive ERCP reserved for the therapeutic procedures such as stenting, sphincterotomy or stone extraction. With a 3 week history of cholestatic jaundice malabsorption of vitamin K is likely to have led to prothrombin time (INR) prolongation and it would be highly dangerous to undertake liver biopsy without first correcting it. The INR should return to normal within 6 hours of a single intravenous dose of vitamin K (5mg).
  • 21. Give a differential diagnosis for a patient presenting with jaundice  Benign recurrent intrahepatic cholestasis  Cholangitis – primary sclerosing  Cholangiocarcinoma  Cholecystitis  Cholelithiasis  Chronic pancreatitis  Cirrhosis  Common bile duct stricture  Extrahepatic malignancy  Genetic non-haemolytic hyperbilirubinaemia (Gilbert’s or Dubin-Johnson syndrome)  Haemolysis  HELLP syndrome (hemolysis, elevated liver tests, and thrombocytopenia)  Hepatitis: viral, bacterial, drug or toxin  Hepatocellular carcinoma  Hyperbilirubinemia of the newborn  Neonate – breast milk jaundice, physiologic jaundice, haemolytic disease  Primary biliary cirrhosis
  • 22. Take a structured history from a patient with jaundice to determine aetiology AmyJoanDice •Presents to ED •Recent onset upper abdominal discomfort •Chills •Yellow sclerae Whatshouldyouask? •Focused HoPC •Medications – liver damage •Alcohol or IVDU •Surgery – biliary, cancer •Transfusions •Pregnancy •Occupation Symptoms? •Character •Sequence •Relievers or conceivers •Associated signs/Sx •Urine/stool •PMHx, FHx
  • 23. Explain the relevance of changes in colour and bilirubin and urobilinogen content of stools and urine in the assessment of jaundice  Dark urine, with green foam upon shaking is caused by bile pigment  excludes hemolysis or a hepatic uptake or conjugating defect of bilirubin metabolism acting alone  Brown stool: haemolysis, mild to moderate hepatocellular  Clay coloured stool: moderate to severe hepatocellular, obstructive  Blood in clay-coloured stool: carcinoma of the pancreas or ampulla of Vater.
  • 24.  A 40 year old woman, who is jaundiced, presents to you with reports of laboratory tests that reveal conjugated hyperbilirubinaemia. Urine bilirubin levels are significantly above normal while urine urobilinogen levels are significantly below normal. Which of the following is most likely cause of her jaundice? A. Blockage of the common bile duct B. Deficiency of glucuronyltransferase C. Gilbert syndrome D. Haemolytic anaemia E. Primary shunt hyperbilirubinaemia
  • 25.  A 40 year old woman, who is jaundiced, presents to you with reports of laboratory tests that reveal conjugated hyperbilirubinaemia. Urine bilirubin levels are significantly above normal while urine urobilinogen levels are significantly below normal. Which of the following is most likely cause of her jaundice? A. Blockage of the common bile duct B. Deficiency of glucuronyltransferase C. Gilbert syndrome D. Haemolytic anaemia E. Primary shunt hyperbilirubinaemia
  • 26. Outline the mechanisms whereby drugs may cause jaundice and give examples of drugs which have each effect  Hepatitis/Hepatotoxicity  Acetaminophen, NSAIDs, Amiodarone, Anabolic steroids, Birth control pills, Chlorpromazine, Erythromycin, Halothan, Methyldopa, Isoniazid, Methotrexate, Statins, Sulfa drugs, Amoxicillin-clavulanate, Anti-epileptics  Cholestasis  Cyclosporine, bosentan, glibenclamide, troglitrazone, rifampicin  Normally, the liver metabolises certain pharma  Susceptible individuals may be poor metabolisers  Hepatocellular uptake and biotransformation results in more water soluble metabolites from lipid soluble drugs  Phase I reactions involve the oxidation, hydroxylation and other reactions mediated by the cytochrome P-450 (CYP) system, particularly CYP3A4. Activity of the cytochrome P-450 system varies greatly among individuals.  Phase II reactions involve esterification reactions that form conjugates with sulfate, glucuronic acid, amino acids or glutathione molecules - enhances detoxification of the compounds.  Can also lead to the production of toxic intermediates  Drug induced cholestasis may happen with high drug concentrations, genetic alterations of enzyme or transporter expression, and/or lower concentrations of anti-oxidants, such as glutathione. Can be caused by direct toxic effects of drugs or their metabolites on liver cells or through by immune mediation
  • 27. Describe how you would investigate a patient with jaundice  Phlebotomy 1) Complete blood cell count and blood smear 1) Haemolysis, reticulocytosis, leukocytosis and neutrophilia, eosinophilia 2) In newborns, blood type and testing for Rh incompatibility (Coomb's) 3) Urinalysis: bilirubin, protein 4) Conjugated and unconjugated (direct and indirect) bilirubin 5) Associated liver function tests (ALP, SGPT and SGOT) 6) Prothrombin and prothrombin precursor, prothrombin time – prognostic 7) Albumin – chronicity 8) Hepatitis A, B, C serology 9) Antibody titres – ANA, delta agent, Epstein-Barr, herpes, cytomegalovirus (leptospirosis, syphilis, entamoeba) 10) Specific markers: ceruloplasmin, transferrin sat, HFE, alpha fetoprotein, protease inhibitors
  • 28. Describe how you would investigate a patient with jaundice  Imaging  Chest X-Ray, Abdomen  Abdominal sonography  CT  Percutaneous transhepatic cholangiography (PTC)  Endoscopic retrograde cholangiopancreatography (ERCP)  Hepatobiliary scintigraphy (HBS)  MRI: metastasis  Biopsy
  • 29. Describe the hepatic origins of ALP, AST and GGT and explain how changes in these may indicate the origin of the jaundice  Alkaline phosphatase  ALP = removes phophates – dephosphorylation  cholestasis, hepatocellular enzyme induction, canalicular injury, bone growth or disease, placenta  Acute lithic biliary obstruction may have aminotransferase levels >500 U/L and normal or mildly elevated ALP  Aminotransferases  AST = amino group catalysis – amino acid metabolism  hepatocellular injury, acute biliary obstruction, coeliac disease, skeletal muscle (AST)  Progression and resolution of hepatocellular injury  AST:ALT >2:1 may indicate alcoholic liver disease  Gamma-glutamyl transpeptidase  GGT catalyses glutathione to an acceptor – detox and glutamate cycle  cholestasis, medications, ethanol, hyperthyroidism, myotonic dystrophy
  • 30.  Jaundice is yellowing of the skin, sclerae and other tissues caused by excess circulating bilirubin. Jaundice is likely to be due to: A. Common bile duct obstruction if the serum aminotransferases are elevated and alkaline phosphatase is low B. Haemolytic disease if plasma albumin is low and globulin high C. Haemolytic disease if prothrombin time is prolonged D. Hepatic disease if plasma albumin is low and serum aminotransferase elevations >500 units E. Hepatic disease if plasma acid phosphatase level is raised
  • 31.  Jaundice is yellowing of the skin, sclerae and other tissues caused by excess circulating bilirubin. Jaundice is likely to be due to: A. Common bile duct obstruction if the serum aminotransferases are elevated and alkaline phosphatase is low B. Haemolytic disease if plasma albumin is low and globulin high C. Haemolytic disease if prothrombin time is prolonged D. Hepatic disease if plasma albumin is low and serum aminotransferase elevations >500 units E. Hepatic disease if plasma acid phosphatase level is raised
  • 32. Give a differential diagnosis for extrahepatic biliary obstruction  Cancer  Cholangiocarcinoma  pancreatic carcinoma  Iatrogenic  surgical injury, stricture, biliary leak  Cholelithiasis  Cholecystitis  Cholangitis
  • 33. Describe the investigation of a patient with extrahepatic biliary obstruction, including endoscopic, radiological and surgical techniques available for treatment/palliation of the underlying condition  Cholelithiasis  Extract lodged stones from the common bile duct tree by performing a procedure called ERCP (endoscopic retrograde cholangiopancreatography).  Strictures  surgical or by interventional endoscopy or radiology with possible further operations to remove stone source  Bile duct cancer  Diagnosed by radiology, furthered by ERCP or PTC and EUS  Resection of the cancer and pathologic exam  Pancreatic cancer  Diagnosed by radiology, multidisciplinary treatment  Resection and biliary tree reconstruction  Cholecystitis; Cholangitis  Cure infection, then cure the underlying cause
  • 34.  Among the following, which is the investigation of choice for evaluation of ONLY the common bile duct? A. CECT abdomen (computed tomography) B. MRCP (magnetic resonance cholangio-pancreatography) C. HIDA (hepatobiliary) scan D. Ultrasonography
  • 35.  Among the following, which is the investigation of choice for evaluation of common bile duct? A. CECT abdomen B. MRCP (magnetic resonance cholangio-pancreatography) C. HIDA (hepatobiliary) scan D. Ultrasonography
  • 36. Describe the symptoms of biliary colic  Pain  1-5 hours of constant severe or dull pain in the epigastrium or RUQ  Peritoneal irritation localises to RUQ  May radiate to scapula or back  Patients move around to seek pain relief  Pain onset hours after meal, often at night, wakes patient  Nausea  Vomiting  Pleuritic pain  Fever
  • 37. Give a differential diagnosis for right upper quadrant pain  Cholecystitis, cholelithiasis, cholangitis, hepatitis, carcinoma of liver, pancreas or biliary tract  Abdominal aneurysm  Diverticular disease  Gastroenteritis  Inflammatory bowel  Mesenteric ischaemia  Myocardial infarction  Pancreatitis  Pleural effusion, right lower lobe pneumonia or tumour  Pregnancy: Eclampsia, Urinary tract infection  Renal calculi  Right colon cancer  Small bowel obstruction  Fractured rib(s)  Spinal root compression
  • 38. Describe the pathogenesis of gallstone formation (cholelithiasis)  Bile is composed mainly of water, bile salts, lecithin (phospholipid) and cholesterol (5%)  Most bile flows into the gallbladder through the cystic duct, while a small amount drains directly to duodenum  Water is removed from bile in the gallbladder (2-5cups/day)  70% of stones are formed of cholesterol  30% are pigmented (black or brown)  Cholesterol is solubilised in micelles  In imbalance between cholesterol and bile salts  cholesterol and calcium bilirubinate sludge  cholesterol crystal precipitation  Predisposing factors: high cholesterol, low gallbladder emptying and absorption, high bilirubin
  • 39. List the risk factors for developing gallstones  4 F’s  Fair  Fat  Female  Fertile  Oral contraceptives or oestrogen replacement  Pregnancy  incidence 5.1% second trimester, 7.9% third, 10.2% 4- 6weeks postpartum  Older age
  • 40. Describe investigations that can be used to confirm the diagnosis of gallstones  Ultrasound – 98% sensitive and specific  Hepatobiliary scintigraphy (HBS) if unclear US  Adjunctive plain radiography, CT, ERCP  US findings for cholecystitis include  Gallstones or sludge  Gallbladder wall thickening (>2-4 mm)  Gallbladder distention (diameter > 4 cm, length >10 cm)  Pericholecystic fluid from perforation or exudate  Air in the gallbladder wall (indicating gangrenous cholecystitis)  Ultrasonographic Murphy sign (86-92% sensitive, 35% specific) - pain when the probe is pushed directly on the gallbladder (not related to breathing) The ultrasound shows gallstones within the gallbladder without evidence of Cholecystitis.
  • 41.  Regarding the diagnosis of cholelithiasis, which of the following is true? A. Offer liver function tests and ultrasound to people with suspected gallstone disease, and to people with abdominal or gastrointestinal symptoms that have been unresponsive to previous management. B. Consider endoscopic retrograde cholangio-pancreatography (ERCP) if ultrasound has not detected common bile duct stones but the: bile duct is dilated and/or liver function test results are abnormal. C. Consider T2 weighted computed tomography (CT) abdomen with enteric contrast if MRCP does not allow a diagnosis to be made. D. Do not refer a person for further investigations if faecal impaction or duodenal atresia is suspected.
  • 42.  Regarding the diagnosis of cholelithiasis, which of the following is true? A. Offer liver function tests and ultrasound to people with suspected gallstone disease, and to people with abdominal or gastrointestinal symptoms that have been unresponsive to previous management. B. Consider endoscopic retrograde cholangio-pancreatography (ERCP) if ultrasound has not detected common bile duct stones but the: bile duct is dilated and/or liver function test results are abnormal. C. Consider T2 weighted computed tomography (CT) abdomen with enteric contrast if MRCP does not allow a diagnosis to be made. D. Do not refer a person for further investigations if the cause is suspected to be faecal impaction or duodenal atresia.
  • 43.  The truth from NICE! 1. Offer liver function tests and ultrasound to people with suspected gallstone disease, and to people with abdominal or gastrointestinal symptoms that have been unresponsive to previous management. 2. Consider MRCP if ultrasound has not detected common bile duct stones but the: bile duct is dilated and/or liver function test results are abnormal. 3. Consider endoscopic ultrasound (EUS) if MRCP does not allow a diagnosis to be made. 4. Refer people for further investigations if conditions other than gallstone disease are suspected.
  • 44. Describe the potential complications of gallstones  Cholecystitis  Gallbladder gangrene in cholecystitis (diabetics, elderly or immunocompromised)  In cholecystitis and/or biliary colic may be cholangitis, sepsis, pancreatitis, hepatitis, and choledocholithiasis  Gallbladder perforation occurs in 10% of cholecystitis  Abscess formation  Free perforation, bile and inflammatory release intra- peritoneally  peritonitis  Gallbladder enteric fistula if perforation occurs next to a hollow organ. Most commonly duodenum.  Gallstones may pass through the fistula into small bowel, and if they >2.5cm can obstruct the ileocecal valve, causing gallstone ileus. Mortality ~20%
  • 45. Discuss the management of a patient with gallstones, including the indications for cholecystectomy  Expectant management (watch and wait)  Asymptomatic patients, 5mm < stones < 3cm  Age for future surgery likelihood  30% at 30, 20% at 50, 15% at 70  Symptomatic antibiotics and NSAIDs (or stronger)  Nonsurgical lithic removal  Lithotripsy <2cm, few  Medication: ursodiol, chenodiol <1.5cm (or contact MTBE)  ERCP removal (endoscopic sphincterotomy)  Surgical cholecystectomy  Laparoscopic or open  No gallstone protection from lowering cholesterol
  • 46.  A 25 year old Hispanic woman, who is 4 months post-partum has pain in the right upper quadrant/ Her laboratory values are as follows: leukocyte count 9.0x103/mm3; total serum bilirubin 0.9mg/dL; serum alkaline phosphatase 100U/L; serum amylase 300 U/L. Which of the following statements regarding her condition is INCORRECT? A. If she undergoes a laparoscopic cholecystectomy, there is an approximately 5% chance that conversion to an open cholecystectomy will be necesasary B. If she undergoes laparoscopic cholecystectomy, she will have a 1.0% to 1.5% chance of having a surgically attributable injury to the common bile duct C. She is likely to have gallstones, which are common in Hispanic women and frequently discovered during or following pregnancy D. She most likely has experienced biliary colic and has passed a gallstone that resulted in mild pancreatitis, rather than having biliary obstruction at this time E. Unless she has cholecystitis, cholelithiasis with symptomatic biliary colic is unlikely
  • 47.  A 25 year old Hispanic woman, who is 4 months post-partum has pain in the right upper quadrant/ Her laboratory values are as follows: leukocyte count 9.0x103/mm3; total serum bilirubin 0.9mg/dL; serum alkaline phosphatase 100U/L; serum amylase 300 U/L. Which of the following statements regarding her condition is INCORRECT? A. If she undergoes a laparoscopic cholecystectomy, there is an approximately 5% chance that conversion to an open cholecystectomy will be necesasary B. If she undergoes laparoscopic cholecystectomy, she will have a 1.0% to 1.5% chance of having a surgically attributable injury to the common bile duct C. She is likely to have gallstones, which are common in Hispanic women and frequently discovered during or following pregnancy D. She most likely has experienced biliary colic and has passed a gallstone that resulted in mild pancreatitis, rather than having biliary obstruction at this time E. Unless she has cholecystitis, cholelithiasis with symptomatic biliary colic is unlikely
  • 48.  E. Unless she has cholecystitis, cholelithiasis with symptomatic biliary colic is unlikely. Symptomatic and asymptomatic cholelithiasis commonly occur in the absence of cholecystitis, an infection of the gallbladder. Cholecystitis can develop from bacteria alone or as a consequence of cystic duct obstruction by a stone. Hispanic women have a high prevalence of biliary disease, including cholelithiasis, which often occurs during and after pregnancy. The laparoscopic approach has become the standard technique for cholecystectomy. The frequency of common bile duct injury is reported to range from 1.0% to 1.5%, or approximately twice that of the open procedure. The reported need to convert to an open cholecystectomy is 5%; to proceed safely, the surgeon should not hesitate to convert to an open technique. The patient is unlikely to have a bile duct obstruction, given her normal serum bilirubin and alkaline phosphatase levels. The elevated serum amylase level suggests that she has experienced biliary colic while passing a stone that resulted in mild pancreatitis.
  • 49. Explain to a patient the function of the gallbladder, how, if at all, its removal may affect them and what steps they can take to minimise such effects  The gallbladder is a pear-shaped organ that rests beneath the right side of the liver.  Its main purpose is to collect and concentrate a digestive liquid (bile) produced by the liver. Bile is released from the gallbladder after eating, aiding digestion. Bile moves through bile ducts into the gut.  Removal of the gallbladder is not associated with any impairment of digestion in most people, but may cause loose stools.  Gallbladder removal is a major operation with postoperative pain, nausea and vomiting up to 2 days.  Recovery is within 7 days, with follow up in 2-3 weeks.
  • 50. Explain to a patient the function of the gallbladder, how, if at all, its removal may affect them and what steps they can take to minimise such effects  Cholecystectomy can relieve the pain and discomfort of gallstones  Cholecystectomy is the only way to prevent gallstones  Contact surgeon if:  Persistent pain, worsening pain  Fever >38.3C  Continuous vomiting  Swelling, redness, bleeding or foul smelling site  No vowel movements 2-3 days post-op  Increasing fibre intake should minimise digestive issues  Balancing to a healthy weight may help
  • 51. Discuss the advantages and disadvantages of laparoscopic surgery  5-7 inch incision versus 4 small openings in the abdomen  Patients usually have minimal post-operative pain  Patients usually experience faster recovery than open gallbladder surgery patients  Most go home same and quicker return to normal  Severe COPD or CHF may not tolerate CO2 pneumoperitoneum  Gallbladder cancer is only operable by open  Common bile duct injury (0.24%)
  • 52.  Regarding laparoscopic cholecystectomy, which of the following is correct? A. Primarily done for cholecystitis in third trimester of pregnancy B. Associated with higher rate of bile duct injury than open cholecystectomy C. Contraindicated in acute cholecystitis D. Safer than open cholecystectomy in patients with cardiorespiratory disease
  • 53.  Regarding laparoscopic cholecystectomy, which of the following is correct? A. Primarily done for cholecystitis in third trimester of pregnancy B. Associated with higher rate of bile duct injury than open cholecystectomy C. Contraindicated in acute cholecystitis D. Safer than open cholecystectomy in patients with cardiorespiratory disease
  • 54. References  MedLine Plus. Updated by: Laura J. Martin, MD, MPH, ABIM Board Certified in Internal Medicine and Hospice and Palliative Medicine, Atlanta, GA. Also reviewed by David Zieve, MD, MHA, Isla Ogilvie, PhD, and the A.D.A.M. Editorial team. Accessed Aug 14 2015. Available from: http://www.nlm.nih.gov/medlineplus/ency/article/002237.htm  John Hopkins Medicine Health Library. Biliary System: Anatomy and Functions Available from: http://www.hopkinsmedicine.org/healthlibrary/conditions/liver_biliary_and_pancreatic_dis orders/biliary_system_anatomy_and_functions_85,P00659/  Available from: http://www.clinbiochem.info/studentlft5.html  Iron storage. Available from: http://library.med.utah.edu/NetBiochem/hi11b.htm  Hyperbilirubinemia and Jaundice. University of Rochester Medical Center Health Encyclopedia. Available from: https://www.urmc.rochester.edu/encyclopedia/content.aspx?ContentTypeID=90&ContentID =P02375  NHS choices. Available from: http://www.nhs.uk/Conditions/Jaundice/PublishingImages/M190040- Jaundice__342x198.JPG  Available from: http://static.guim.co.uk/sys- images/Guardian/About/General/2010/10/20/1287571854392/Homer-Simpson-006.jpg  Available from: http://www.ncbi.nlm.nih.gov/books/NBK413/
  • 55. References  Acute Cholecystitis and Biliary Colic. Medscape. Author: Peter A D Steel, MA, MBBS; Chief Editor: Barry E Brenner. Accessed Aug 15 2015. Available from: http://emedicine.medscape.com/article/1950020-overview  http://www.uphs.upenn.edu/surgery/Education/medical_students/links/Jaundice.pdf  http://www.aldersonfuneralhomes.com/tribute-images/666/High/Dice-2C_Joan_T-.jpg  http://www.nlm.nih.gov/medlineplus/ency/article/000226.htm  http://www.medscape.com/viewarticle/710045_3  http://www.cpmc.org/advanced/liver/patients/topics/bileduct-profile.html  http://umm.edu/health/medical/reports/articles/gallstones-and-gallbladder-disease  http://www.sages.org/publications/patient-information/patient-information-for-laparoscopic- gallbladder-removal-cholecystectomy-from-sages/  http://www.mayoclinic.org/tests-procedures/cholecystectomy/basics/what-you-can-expect/prc- 20013253  https://www.facs.org/~/media/files/education/patient%20ed/cholesys.ashx  https://books.google.co.il/books?id=_1j3LnyNAiIC&printsec=frontcover&source=gbs_ge_summary_r& cad=0#v=onepage&q=6.33&f=false  http://www.netmedicos.com/?surgery/mcqs_3/page/4/  http://www.cirse.org/files/files/EBIR/MCQ/EBIR_MCQ.pdf  http://www.doctorsintraining.com/mkt/qa/  http://www.nice.org.uk/guidance/cg188/resources/guidance-gallstone-disease-pdf
  • 56.  When performing biliary drainage and stenting as a palliative treatment for malignant obstructive jaundice. Which kind of equipment is preferable? A. Plastic retrievable endoprostheses B. Self-expanding metallic stents C. Balloon-expanding metallic stents D. Covered self-expanding stents
  • 57.  When performing biliary drainage and stenting as a palliative treatment for malignant obstructive jaundice. Which kind of equipment is preferable? A. Plastic retrievable endoprostheses B. Self-expanding metallic stents C. Balloon-expanding metallic stents D. Covered self-expanding stents